Вы находитесь на странице: 1из 21

Construction of Insurance Contracts- Art. 1377. The interpretation of obscure words or 3. Qua Chee Gan v.

Qua Chee Gan v. Law Union and Rock Insurance Co.


stipulations in a contract shall not favor the party who caused the obscurity.
FACTS: Qua CheeGan owns four warehouses in Albay. He was using these warehouses
1. Calanoc v. Court of Appeals for the storage of stocks of copra and hemp. All warehouses were insured by Law Union
and Rock Insurance for the amount of P370,000.00. The insurance states that Qua
The circumstance that he was a mere watchman and had no duty to heed the call of CheeGan should install 11 hydrants in the warehouses’ premises. Qua CheeGan installed
Atty. Ojeda should not be taken as a capricious desire on his part to expose his life to only two, but Law Union nevertheless went on with the insurance policy and collected
danger considering the fact that the place he was in duty-bound to guard was only a premiums from Qua CheeGan. The insurance contract also provides that “oil” should
block away. In volunteering to extend help under the situation, he might have thought, not be stored within the premises of the warehouses.
rightly or wrongly, that to know the truth was in the interest of his employer it being a
matter that affects the security of the neighborhood. No doubt there was some risk In 1940, three of the warehouses were destroyed by fire. The damage caused
coming to him in pursuing that errand, but that risk always existed it being inherent in amounted to P398,562.81. Qua CheeGan demanded insurance pay from Law Union but
the position he was holding. He cannot therefore be blamed solely for doing what he the latter refused claiming violation of warranties and conditions, filing of fraudulent
believed was in keeping with his duty as a watchman and as a citizen. And he cannot be claims, and that the fire had been deliberately caused by Qua CheeGan. Law Union in
considered as making an arrest as an officer of the law, as contended, simply because he fact sued Qua CheeGan for Arson. Qua CheeGan was acquitted by the trial court in the
went with the traffic policeman, for certainly he did not go there for that purpose nor arson case. He then demanded that Law Union pay up. This time, Law Union averred
was he asked to do so by the policeman. While as a general rule “the parties may limit that the insurance contract is void because Qua CheeGan failed to install 11 hydrants;
the coverage of the policy to certain particular accidents and risks or causes of loss, and and that gasoline was found in one of the warehouses.
may expressly except other risks or causes of loss therefrom” (45 C. J. S. 781-782),
however, it is to be desired that the terms and phraseology of the exception clause be ISSUES:
clearly expressed so as to be within the easy grasp and understanding of the insured, for
if the terms are doubtful or obscure the same must of necessity be interpreted or 1) Whether or not the insurance company can void the policies it had issued
resolved aganst the one who has caused the obscurity. (Article 1377, new Civil Code) 2) Whether or not the insured violated the "Hemp Warranty" provisions of the policy
And so it has bene generally held that the “terms in an insurance policy, which are against the storage of gasoline.
ambiguous, equivacal, or uncertain . . . are to be construed strictly and most strongly
against the insurer, and liberally in favor of the insured so as to effect the dominant RULING:
purpose of indemnity or payment to the insured, especially where a forfeiture is
involved” (29 Am. Jur., 181), and the reason for this rule is that he “insured usually has 1) No. Law Union cannot exempt itself from paying Qua CheeGan.
no voice in the selection or arrangement of the words employed and that the language
of the contract is selected with great care and deliberation by experts and legal advisers Where the insurer, at the time of the issuance of a policy of insurance, has
employed by, and acting exclusively in the interest of, the insurance company.” knowledge of existing facts which, if insisted on, would invalidate the contract from its
very inception, such knowledge constitutes a waiver of conditions in the contract
2. Biagtan v. Insular Life inconsistent with the facts, and the insurer is estopped thereafter from asserting the
breach of such conditions. The appellant insurance company is barred by waiver (or
It should be noted that the exception in the accidental benefit clause invoked by the rather estoppel) to claim violation of the so-called fire hydrants warranty, for the reason
appellant does not speak of the purpose — whether homicidal or not — of a third party that knowing fully all that the number of hydrants demanded therein never existed from
in causing the injuries, but only of the fact that such injuries have been “intentionally” the very beginning, the insurance company nevertheless issued the policies in question
inflicted — this obviously to distinguish them from injuries which, although received at subject to such warranty, and received the corresponding premiums. The appellant
the hands of a third party, are purely accidental. This construction is the basic idea company so worded the policies that while exacting the greater number of fire hydrants
expressed in the coverage of the clause itself, namely, that “the death of the insured and appliances, it kept the premium discount at the minimum of 2 1/2%, thereby giving
resulted directly from bodily injury effected solely through external and violent means the insurance company a double benefit. Such abnormal treatment of the insured
sustained in an accident … and independently of all other causes.” But where a gang of strongly points at an abuse of the insurance company's selection of the words and terms
robbers enter a house and coming face to face with the owner, even if unexpectedly, of the contract, over which it had absolute control. Moreover, taking into account the
stab him repeatedly, it is contrary to all reason and logic to say that his injuries are not well known rule that ambiguities or obscurities must be strictly interpreted against the
intentionally inflicted, regardless of whether they prove fatal or not. party that caused them, the "memo of warranty" invoked by appellant bars the latter
As it was, in the present case they did prove fatal, and the robbers have been accused from questioning the existence of the appliances called for in the insured premises.
and convicted of the crime of robbery with homicide.
2) No. The ambiguity must be held strictly against the insurer and liberally in favor of The court ruled in this manner because in the insurance policy of the petitioner
the insured, specially to avoid a forfeiture. there is no mention of mere injury short of amputation of the hand as equating with the
definition of “loss of either hand” since in the policy itself it defines loss as: “…the loss a
Insurance is, in its nature, complex and difficult for the layman to understand. hand shall mean the loss by amputation through the bones of the wrist…” The courts
Policies are prepared by experts who know and can anticipate the hearing and possible will only hazard an interpretation of such insurance policies if there is ambiguity with its
complications of every contingency. So long as insurance companies insist upon the use provisions; in this case, the policy is clear on what is compensable, therefore the courts
of ambiguous, intricate and technical provisions, which conceal rather than frankly must apply the ordinary meaning of the provision, which is that the definition of loss is
disclose, their own intentions, the courts must, in fairness to those who purchase limited to amputation.
insurance, construe every ambiguity in favor of the insured. Appellee admitted that
there were 36 cans of gasoline in the building designed. However, gasoline is not 5. De la Cruz v. Capital Insurance
specifically mentioned among the prohibited articles listed in the so-called "hemp
warranty." The cause relied upon by the insurer speaks of "oils", and is uncertain FACTS: Eduardo de la Cruz, employed as a mucker in the Itogon-Suyoc Mines, Inc. in
because, "Oils" usually mean "lubricants" and not gasoline or kerosene. If the company Baguio, was the holder of an accident insurance policy "against death or disability
intended to rely upon a condition of that character, it ought to have been plainly caused by accidental means". On January 1, 1957: For the celebration of the New Year,
expressed in the policy. The contract of insurance is one of perfect good faith the Itogon-Suyoc Mines, Inc. sponsored a boxing contest for general entertainment
(uferrimalfidei) not for the insured alone, but equally so for the insurer; in fact, it is mere wherein Eduardo, a non-professional boxer participated. In the course of his bout with
so for the latter, since its dominant bargaining position carries with it stricter another non-professional boxer of the same height, weight, and size, Eduardo slipped
responsibility. and was hit by his opponent on the left part of the back of the head, causing Eduardo to
fall, with his head hitting the rope of the ring. He was brought to the Baguio General
Also, the gasoline kept in Bodega No. 2 was only incidental to his business, Hospital the following day. He died due to hemorrhage, intracranial. Simon de la Cruz,
being no more than a customary 2 day's supply for the five or six motor vehicles used the father of the insured and who was named beneficiary under the policy, thereupon
for transporting of the stored merchandise. "It is well settled that the keeping of filed a claim with the insurance company. The Capital Insurance and Surety co.,
inflammable oils on the premises though prohibited by the policy does not void it if such inc denied stating that the death caused by his participation in a boxing contest was not
keeping is incidental to the business." accidental. Appellant argued that to be considered within the protection of the policy,
what is required to be accidental is the means that caused or brought the death and not
4. Ty v. First National Surety Assurance the death itself.

FACTS: 2 months prior to December 24, 1953: Diosdado C. Ty, employed as operator ISSUE: Whether the cause of death was accident
mechanic foreman in the Broadway Cotton Factory insured himself in 18 local insurance
companies with Broadway Cotton Factory as his beneficiary. December 24, 1953: fire RULING: Eduardo slipped, which was unintentional. The terms "accident" and
broke out at the Broadway Cotton Factory where Ty, fighting his way out, injured his left "accidental" as used in insurance contracts, have not acquired any technical
hand by a heavy object. He was brought to the Manila Central University hospital, and meaning and are construed by the courts in their ordinary and common acceptation
after receiving first aid there, he went to the National Orthopedic Hospital for treatment happen by chance or fortuitously, without intention and design, and which is
of his injuries. His injuries caused temporary total disability on his left hand so he filed a unexpected, unusual, and unforeseen event that takes place without one's foresight or
claim against all defendants who rejected the claim reasoning that there it was not expectation event that proceeds from an unknown cause, or is an unusual effect of a
covered in his policy because there was no severance of amputation of the left hand. known cause and, therefore, not expected where the death or injury is not the natural
or probable result of the insured's voluntary act, or if something unforeseen occurs in
ISSUE: Can injuries that prevent the appellant from performing his work or labor the doing of the act which produces the injury, the resulting death is within the
necessary in the pursuance of his occupation or business be equated with “loss of left protection of policies insuring against death or injury from accident while the
hand” for the purpose of the latter claiming from his insurance policies? participation of the insured in the boxing contest is voluntary, the injury was sustained
when he slid, giving occasion to the infliction by his opponent of the blow that threw
RULING: The agreement contained in the insurance policies is the law between the him to the ropes of the ring is not. The fact that boxing is attended with some risks of
parties. As the terms and conditions are clear, express and specific that only amputation external injuries does not make any injuries received in the course of the game not
of the left hand should be considered as a loss thereof, an interpretation that would accidental. In boxing as in other equally physically rigorous sports, such as basketball or
include the mere fracture or other temporary disability not covered by the policies baseball, death is not ordinarily anticipated to result. If, therefore, it ever does, the
would certainly be unwarranted. injury or death can only be accidental or produced by some unforeseen happening or
event as what occurred in this case.
Reyes, who cannot read and write, who has never passed any examination for drivers,
6. Taurus Taxi v. Capital Insurance and has not applied for a license from the duly constituted government agency
entrusted with the duty of licensing drivers, cannot be considered an authorized driver
FACTS: (1) Alfredo Monje was employed as a taxi driver of the Taurus Taxi. (2) On
December 6, 1962, the taxi he was driving collided with a Transport Taxicab at the AUTHORIZED DRIVER:
intersection of Old Sta. Mesa and V. Mapa Streets, Manila, resulting in his death. (3) Any of the following: (a) The insured; (b) Any person driving on the Insured's order or
Commercial Vehicle Comprehensive Policy was subsisting at the time of the accident. with his permission, provided that the person driving is permitted in accordance with
The Policy states that the amount for which each passenger, including the driver, is licensing laws or regulations to drive the motor vehicle covered by this Policy, or has
insured is P5,000.00. (4) The Indorsement No. 1 which forms part of the policy was given been so permitted and is not disqualified by order of a court of law or by reason of any
to Petitoner Felicitas and his children. (5) Taurus Taxi made representation "for the enactment or regulation from driving such Motor Vehicle.
payment of the insurance benefit which corresponds to the wife and children".
Defendant refused to pay. (6) DEFENDANTS CONTENTION: that in view of the fact that ISSUE: Whether Domingo Reyes was an authorized driver
the deceased Alfredo Monje was entitled to indemnity under ANOTHER INSURANCE
POLICY issued by Ed. A. Keller Co., Ltd., the heirs of the said deceased are not entitled to HELD: YES. A driver’s license that bears all the earmarks of a duly issued license is a
indemnity under the insurance policy issued by appellant for the reason that the latter public document which is presumed genuine. The presumption of genuineness in its
POLICY contains a stipulation that "the company will indemnify any authorized driver issuance is not disproved by a mere certification by an agency of the Motor Vehicles
provided that such authorized driver is not entitled to indemnity under any other policy. Office that it did not issue the license in question because said office may have been
mistaken or that said license was issued by another agency, and the person who issued
HELD: In the case at bar, the insurance policy contained a prohibition to the effect that the certification was not placed on the witness stand. Under section 24 of the Revised
any "authorized driver of plaintiff Taurus Taxi Co., Inc. should not be entitled to any Motor Vehicles Law, Act 3992 of the Philippine Legislature, as amended by RA Nos. 587,
idemnity under any other policy." However, the Workmen's Compensation Act explicitly 1204 and 2363, a driver’s license may be issued with or without previous examination in
requires that an employee suffering any injury or death arising out of or in the course of the discretion of the Motor Vehicles Office officials.
employment be compensated. Under the circumstances, the fact that the deceased was
paid his workman's compensation from another policy, should not defeat the right to 8. Sun Life Insurance v. Court of Appeals
recover under such insurance policy despite the prohibition mentioned. It is too well-
settled to need the citation of authorities that what the law requires (as the Workmen's FACTS: The petitioner issued Personal Accident Policy No. 05687 to Felix Lim, Jr. with a
Compensation Act) enters into and forms part of every contract. The fulfillment of the face value of P200,000.00. Two months later, he was dead with a bullet wound in his
statutory obligation under said Act cannot be the basis for evading the clear, explicit and head. As beneficiary, his wife Nerissa Lim sought payment on the policy but her claim
mandatory terms of a policy. was rejected. The petitioner agreed that there was no suicide. It argued, however that
there was no accident either. Pilar Nalagon, Lim's secretary, was the only eyewitness to
From its own version, defendant-appellant would seek to escape liability on the plea his death. It happened in the evening, after his mother's birthday party. According to
that the workman's compensation to which the deceased driver was rightfully entitled Nalagon, Lim was in a happy mood (but not drunk) and was playing with his handgun,
was settled by the employer through a policy issued by another insurance firm. What from which he had previously removed the magazine. As she watched television, he
was paid therefore was not indemnity but compensation. stood in front of her and pointed the gun at her. She pushed it aside and said it might he
loaded. He assured her it was not and then pointed it to his temple. The next moment
A contract of insurance couched in language chosen by the insurer is, if open to the there was an explosion and Lim slumped to the floor. He was dead before he fell. The
construction contended for by the insured, to be construed most strongly, or strictly, widow sued the petitioner in the Regional Trial Court of Zamboanga City and was
against the insurer and liberally in favor of the contention of the insured, which means sustained. The petitioner also cites one of the four exceptions provided for in the
in accordance with the rule contra proferentem. insurance contract and contends that the private petitioner's claim is barred by such
provision. It is there stated: i) The insured person attempting to commit suicide or
7. CCC Insurance v. Court of Appeals willfully exposing himself to needless peril except in an attempt to save human life. The
petitioner maintains that by the mere act of pointing the gun to hip temple, Lim had
FACTS: Carlos F. Robes insured with the CCC Insurance Corporation his Dodge Kingsway willfully exposed himself to needless peril and so came under the exception.
car against loss or damage through accident for an amount not exceeding P8,000. On
June 25 1961, Carlos' driver Domingo Reyes met a vehicular collision along Rizal Avenue
Extension, Potrero, Malabon, Rizal. CCC Insurance Corporation denied his claim ISSUE: Whether the insured willfully exposed himself to needless peril and thus
reasoning that the driver was not an "authorized driver". removed himself from the coverage of the insurance policy.
RULING: YES. It should be noted at the outset that suicide and willful exposure to FCP filed a complaint against private respondents, who in turn filed an amended third
needless peril are in pari materia because they both signify a disregard for one's life. The party complaint against petitioner Perla. The trial court rendered a decision in favor of
only difference is in degree, as suicide imports a positive act of ending such life whereas FCP and DISMISSAL of the Third-Party Complaint filed against Perla. Private respondents
the second act indicates a reckless risking of it that is almost suicidal in intent. appealed the same to the Court of Appeals, which reversed said decision holding that
private respondents did not violate the insurance contract because the authorized
That posture is arguable. But what is not is that, as the secretary testified, Lim had driver clause is not applicable to the "Theft" clause of said Contract.
removed the magazine from the gun and believed it was no longer dangerous. He
expressly assured her that the gun was not loaded. It is submitted that Lim did not ISSUE: Whether the “authorized driver clause” is applicable in this case.
willfully expose himself to needless peril when he pointed the gun to his temple because
the fact is that he thought it was not unsafe to do so. The act was precisely intended to
RULING: No. The comprehensive motor car insurance policy issued by petitioner Perla
assure Nalagon that the gun was indeed harmless. Lim was unquestionably negligent
undertook to indemnify the private respondents against loss or damage to the car by,
and that negligence cost him his own life. But it should not prevent his widow from
among others, by fire, external explosion, self-ignition or lightning or burglary,
recovering from the insurance policy he obtained precisely against accident. There is
housebreaking or theft. Where a car is admittedly, as in this case, unlawfully and
nothing in the policy that relieves the insurer of the responsibility to pay the indemnity
wrongfully taken without the owner's consent or knowledge, such taking constitutes
agreed upon if the insured is shown to have contributed to his own accident. Indeed,
theft, and, therefore, it is the "THEFT"' clause, and not the "AUTHORIZED DRIVER" clause
most accidents are caused by negligence.
that should apply. Theft is an entirely different legal concept from that of accident. Theft
is committed by a person with the intent to gain or, to put it in another way, with the
9. Perla Cia. De Seguros v. Court of Appeals
concurrence of the doer's will. On the other hand, accident, although it may proceed or
result from negligence, is the happening of an event without the concurrence of the will
FACTS: Private respondents spouses Herminio and Evelyn Lim executed a promissory of the person by whose agency it was caused. Clearly, the risk against accident is distinct
note in favor Supercars, Inc. in the sum of P77,940.00, and secured by a chattel from the risk against theft. The "authorized driver clause" in a typical insurance policy is
mortgage over a brand new red Ford Laser which is registered under the name of in contemplation or anticipation of accident in the legal sense in which it should be
private respondent Herminio Lim and insured with the petitioner Perla Compania de understood, and not in contemplation or anticipation of an event such as theft. In the
Seguros, Inc. Supercars, Inc., with notice to private respondents spouses, assigned to present case. The loss of the insured vehicle did not result from an accident where
petitioner FCP Credit Corporation (FCP for brevity) its rights, title and interest on said intent was involved; the loss in the present case was caused by theft, the commission of
promissory note and chattel mortgage as shown by the Deed of Assignment. which was attended by intent. It is worthy to note that there is no causal connection
between the possession of a valid driver's license and the loss of a vehicle.
Private respondent filed a claim for loss with the petitioner Perla but said claim was
denied on the ground that Evelyn Lim, who was using the vehicle before it was FCP: This Court agrees with petitioner FCP that private respondents are not relieved of
carnapped, was in possession of an expired driver's license at the time of the loss of said their obligation to pay the former the installments due on the promissory note on
vehicle which is in violation of the authorized driver clause of the insurance policy, account of the loss of the automobile. The chattel mortgage constituted over the
which states, to wit: automobile is merely an accessory contract to the promissory note. Being the principal
contract, the promissory note is unaffected by whatever befalls the subject matter of
AUTHORIZED DRIVER: the accessory contract. Therefore, the unpaid balance on the promissory note should be
paid, and not just the installments due and payable before the automobile was
Any of the following: (a) The Insured (b) Any person driving on the carnapped.
Insured's order, or with his permission. Provided that the person
driving is permitted, in accordance with the licensing or other laws
or regulations, to drive the Scheduled Vehicle, or has been 10. Fortune Insurance v. Court of Appeals
permitted and is not disqualified by order of a Court of Law or by
reason of any enactment or regulation in that behalf. FACTS: Producers Bank was insured by Fortune Insurance. Producers Bank filed against
Fortune Insurance a complaint for recovery of the sum of P725,000.00 under the policy
Perla, however, denied private respondents' claim. Consequently, petitioner FCP issued by Fortune. The sum was allegedly lost during a robbery of Producer’s armored
demanded that private respondents pay the whole balance of the promissory note or to vehicle while it was in transit to transfer the money from its Pasay City Branch to its
return the vehicle but the latter refused. head office in Makati.
The said armored vehicle was robbed by its driver Benjamin Magalong and security In short, for these particular tasks, the three actedas agents of Producers.
guard Saturnino Atiga tasked to man the same. Both of them are not Producers Bank’s A “representative” is defined as one who represents or stands inthe place of another;
“employees” but were merely assigned by and affiliated with PRC Management Systems one who represents others or another in a special capacity, as anagent, and is
and Unicorn Security Services. Fortune Insurance refused to pay the amount as the loss, interchangeable with “agent.”
according to it, is excluded from the coverage of the insurance policy. “General
Exceptions” provides: The company shall not be liable under this policy in report of x x x
Business of Insurance
(b) any loss caused by any dishonest, fraudulent or criminal act of the insured or any
officer, employee, partner, director, trustee or authorized representative of the Insured
1. White Gold Marine Service v. Pioneer Insurance
whether acting alone or in conjunction with others…” Producers Bank opposed the
contention of Fortune Insurance and contends that Atiga and Magalong are not its
FACTS: White Gold, petitioner, procured a protection and indemnity for its vessel from
officer, employee, trustee, or authorized representative at the time of the robbery.
the Steamship Mutual Underwriting Association through Pioneer Insurance and Security
According to Fortune Insurance, when Producers commissioned a guard and a driver to
Corporation. Subsequently, White Gold was issued a Certificate of Entry and
transfer its funds from one branch to another, they effectively and necessarily became
Acceptance. When petitioner failed to fully pay its account, Steamship Mutual refused
its authorized representatives in the care and custody of the money. Assuming that they
to renew the coverage. Steamship thereafter filed a case of collection of sum of money
could not be considered authorized representatives, they were, nevertheless,
for the unpaid balance of the petitioner while the latter filed before the Insurance
employees of Producers.
Commissioner a case against Steamship for violating Sections 186 and 187 of the
Insurance Code, while Pioneer violated Sections 299,] 300 and 301 in relation to
ISSUE: Whether Magalong and Atiga qualify as employees or authorized representatives Sections 302 and 303, thereof. The Insurance Commissioner dismissed the complaint
of Producers under paragraph (b) of the general exceptions clause of the insurance and said that there is no need for the Steamship Mutual to procure license because it
policy as to exempt Fortune Insurance from liability to pay Producers Bank under said was not engage in insurance business and was only a protection and indemnity club.
policy? Likewise, it ruled that Pioneer need not secure another license as an insurance agent
and/or a broker of Steamship Mutual because it was not engaged in insurance business
and Pioneer already had a license hence procurement of separate license as an
HELD: Yes. Employer-employee relationship depends upon four standards: (1) the
insurance agent would only be superfluous. CA affirmed the decision of Insurance
manner of selectionand engagement of the putative employee; (2) the mode of
Commissioner.
payment of wages; (3) thepresence or absence of a power to dismiss; and (4) the
presence and absence of a powerto control the putative employee’s conduct The power
of control over Magalong and Atiga was vested in and exercised by Producers Bank; ISSUE: Whether or not Steamship Mutual, a P & I Club, is engaged in the insurance
hence, an “employer-employee” relationship exists between Magalong and Atigaand business in the Philippines (2) Does Pioneer need a license as an insurance agent/broker
Producers Bank. PRC Management System and Unicorn Security Services are but “labor- for Steamship Mutual?
only” contractors(not employers) under Article 106 of the Labor Code which provides:
“There is “labor-only” contracting where the person supplying workers to an employer
RULING: For the first issue, the court answered positively. Section 2(2) of the Insurance
does not have substantial capital or investment in the form of tools, equipment,
Code enumerates what constitutes doing an insurance business or transacting an
machineries, work premises, among others, and the workers recruited and placed by
insurance business. These are: (a) making or proposing to make, as insurer, any
such persons are performing activities which are directly related to the principal
insurance contract; (b) making, or proposing to make, as surety, any contract of
business of such employer. In such cases, the person or intermediary shall be
suretyship as a vocation and not as merely incidental to any other legitimate business or
considered merely as an agent of the employer who shall be responsible to the workers
activity of the surety; (c) doing any kind of business, including a reinsurance business,
in the same manner and extent as if the latter were directly employed by
specifically recognized as constituting the doing of an insurance business within the
him.”Magalong and Atiga were, in respect of the transfer of Producers Bank’s money
meaning of this Code; (d) doing or proposing to do any business in substance equivalent
from its Pasay City branch to its head office in Makati, its “authorized
to any of the foregoing in a manner designed to evade the provisions of this Code. The
representatives” who served as such with its teller Maribeth Alampay. However viewed,
test to determine if a contract is an insurance contract or not, depends on the nature of
Producers entrusted the three with the specific duty to safely transfer the money to its
the promise, the act required to be performed, and the exact nature of the agreement
head office, with Alampay to be responsible for its custody in transit; Magalong to drive
in the light of the occurrence, contingency, or circumstances under which the
the armored vehicle which would carry the money; and Atiga to provide the needed
performance becomes requisite. It is not by what it is called. In particular, a marine
security for the money, the vehicle, and his two other companions.
insurance undertakes to indemnify the assured against marine losses, such as the losses
incident to a marine adventure.
Relatedly, a mutual insurance company is a cooperative enterprise where the members On appeal, the decision of the trial court was affirmed but deleted all awards for
are both the insurer and insured. In it, the members all contribute, by a system of damages and absolved petitioner Reverente. Hence, this petition for review raising the
premiums or assessments, to the creation of a fund from which all losses and liabilities primary argument that a health care agreement is not an insurance contract; hence the
are paid, and where the profits are divided among themselves, in proportion to their “incontestability clause” under the Insurance Code does not apply.
interest.
ISSUES: Whether or not the health care agreement is an insurance contract.
Additionally, mutual insurance associations, or clubs, provide three types of coverage,
RULING: NO. Section2 (1)of the Insurance Code defines a contract of insurance as an
namely, protection and indemnity, war risks, and defense costs. A P & I Club is a form of
agreement whereby one undertakes for a consideration to indemnify another against
insurance against third party liability, where the third party is anyone other than the P &
loss, damage, or liability arising from an unknown or contingent event. Section 3 of the
I Club and the members. By definition then, Steamship Mutual as a P & I Club is a mutual
Insurance Code states that any contingent or unknown event, whether past or future,
insurance association engaged in the marine insurance business. The records reveal
which my damnify a person having an insurable against him, may be insured against.
Steamship Mutual is doing business in the country albeit without the requisite
Every person has an insurable interest in the life and health of himself. Section 10
certificate of authority mandated by Section 187[20] of the Insurance Code. It maintains
provides that every person has an insurable interest in the life and health (1) of himself,
a resident agent in the Philippines to solicit insurance and to collect payments in its
of his spouse and of his children.
behalf. We note that Steamship Mutual even renewed its P & I Club cover until it was
cancelled due to non-payment of the calls. Thus, to continue doing business here,
The insurable interest of respondent’s husband in obtaining the health care agreement
Steamship Mutual or through its agent Pioneer, must secure a license from the
was his own health. The health care agreement was in the nature of non-life insurance,
Insurance Commission. For the second issue, the court ruled that although Pioneer is
which is primarily a contract of indemnity. Once the member incurs hospital, medical or
already licensed as an insurance company, it needs a separate license to act as
any other expense arising from sickness, injury or other stipulated contingent, the
insurance agent for Steamship Mutual.
health care provider must pay for the same to the extent agreed upon under the
contract. Petitioner alleges that respondent was not the legal wife of the deceased
2. Philamcare Health System v. Court of Appeals member considering that at the time of their marriage, the deceased was previously
married to another woman who was still alive. The health care agreement is in the
FACTS: Ernani Trinos applied for a health care coverage with Philamcare Health Systems, nature of a contract of indemnity. Hence, payment should be made to the party who
Inc. To the question ‘Have you or any of your family members ever consulted or been incurred the expenses. It is not controverted that respondent paid all the hospital and
treated for high blood pressure, heart trouble, diabetes, cancer, liver disease, asthma or medical expenses. She is therefore entitled to reimbursement. The records adequately
peptic ulcer?’, Ernani answered ‘No’. Under the agreement, Ernani is entitled to avail of prove the expenses incurred by respondent for the deceased’s hospitalization,
hospitalization benefits and out-patient benefits. The coverage was approved for a medication and the professional fees of the attending physicians.
period of one year from March 1, 1988 to March 1, 1989. The agreement was however
extended yearly until June 1, 1990 which increased the amount of coverage to a The Contract of Insurance: (Read Insurance Code Sec. 1-10)
maximum sum of P75,000 per disability. During the period of said coverage, Ernani Filipinas Via. De Seguros v. Christern Huenfeld
suffered a heart attack and was confined at the Manila Medical Center (MMC) for one
month. While in the hospital, his wife Julita tried to claim the benefits under the health FACTS: Christern, Huenefeld and Company, a German company, obtained a fire
care agreement. However, the Philamcare denied her claim alleging that the agreement insurance policy from Filipinas Compañia for the merchandise contained in a building
was void because Ernani concealed his medical history. Doctors at the MMC allegedly located in Binondo, Manila in the sum of P100,000. Filipinas Compañia is an American
discovered at the time of Ernani’s confinement that he was hypertensive, diabetic and controlled company. The building and the insured merchandise were burned during the
asthmatic, contrary to his answer in the application form. Thus, Julita paid for all the Japanese occupation. Christern filed its claim amounting to P92,650.00 but Filipinas
hospitalization expenses. After Ernani was discharged from the MMC, he was attended Compañia refused to pay alleging that Christern is a corporation whose majority
by a physical therapist at home. Later, he was admitted at the Chinese General Hospital. stockholders are Germans and that during the Japanese occupation, America declared
Due to financial difficulties, however, respondent brought her husband home again. In war against Germany hence the insurance policy ceased to be effective because the
the morning of April 13, 1990, Ernani had fever and was feeling very weak. Respondent insured has become an enemy. Filipinas Compañia was eventually ordered to pay
was constrained to bring him back to the Chinese General Hospital where he died on the Christern as ordered by the Japanese government.
same day. Julita filed an action for damages and reimbursement of her expenses plus
moral damages attorney’s fees against Philamcare and its president, Dr. Benito ISSUE: Whether or not Christern, Huenefeld and Co is entitled to receive the proceeds
Reverente. The Regional Trial court or Manila rendered judgment in favor of Julita. from the insurance claim.
RULING: NO. There is no question that majority of the stockholders of Christern were Connecticut Rule: The first holds the view that "there are two elements in the
German subjects. This being so, Christern became an enemy corporation upon the consideration for which the annual premium is paid — First, the mere protection for the
outbreak of the war between the United States and Germany. The Philippine Insurance year, and second, the privilege of renewing the contract for each succeeding year by
Law (Act No. 2427, as amended,) in Section 8, provides that “anyone except a public paying the premium for that year at the time agreed upon. According to this view of the
enemy may be insured.” It stands to reason that an insurance policy ceases to be contract, the payment of premiums is a condition precedent, the non-performance
allowable as soon as an insured becomes a public enemy. The respondent having would be illegal necessarily defeats the right to renew the contract."
become an enemy corporation on December 10, 1941, the insurance policy issued in its
favor on October 1, 1941, by the petitioner had ceased to be valid and enforceable, and New York Rule: The second rule, apparently followed by the greater number of
since the insured goods were burned after December 10, 1941, and during the war, the decisions, hold that "war between states in which the parties reside merely suspends
respondent was not entitled to any indemnity under said policy from the petitioner. the contracts of the life insurance, and that, upon tender of all premiums due by the
However, elementary rules of justice (in the absence of specific provision in the insured or his representatives after the war has terminated, the contract revives and
Insurance Law) require that the premium paid by the respondent for the period covered becomes fully operative."
by its policy from December 11, 1941, should be returned by the petitioner.
United States Rule: The United States rule declares that the contract is not merely
1. Constantino v. Asia Life Insurance
suspended, but is abrogated by reason of non-payments is peculiarly of the essence of
the contract. It additionally holds that it would be unjust to allow the insurer to retain
FACTS: FIRST CASE: Respondent Corporation was paid P 176.04 as annual premium by
the reserve value of the policy, which is the excess of the premiums paid over the actual
Arcadio Constantino in exchange for policy no. 93212 on 1941 for P 3,000 which lasted
risk carried during the years when the policy had been in force.
for 20 years. Petitioner Paz Constantino was made beneficiary. However after the first
payment, no further premiums were made. Thereafter the insured died on 1944. Later,
due to the war (Japanese occupation) Respondent Corporation had to close down its 2. Sales de Gonzaga v. Crown Life Insurance
branch in the country.
FACTS: September 26, 1939: Crown Life Insurance Co. whose home office is based in
SECOND CASE: Similarly, Respondent Corporation issued on 1938 another insurance Toronto, Canada issued to Ramon Gonzaga through its branch office in Manila a 20-
policy no. 78145 for Spouses Ruiz and Peralta also for P 3,000, lasting for 20 years. year endowment policy for P15,000 which had an annual premium of P591. Payment
Regular payments were made however due also to the war, it became impossible to was only until September 6, 1941 because of the outbreak of the war since Crown is an
transact further payments. The insured nevertheless was able to borrow P 234 from the enemy corp. order to be closed during the Japanese occupation. However, despite that
policy. Ruiz died on 1945. Peralta was the beneficiary. In both cases the plaintiffs it offered a privilege to accept premium payments in the place of its employee in Ermita
demanded payment but was refused due to Respondent Corporation’s refusal on the but of which Gonzaga did not avail. Through the automatic premium loan clause, it
ground of non-payment of the premiums. The lower court favored Respondent. continued until June 12, 1943. On May 1, 1945: It reopened but still Gonzaga did not pay
although there was a reinstatement clause providing certain conditions within three
ISSUES: Whether or not the beneficiaries are entitled to recover the amount insured years from the date of lapse on application of the insured. On June 27, 1945: Gonzaga
despite non-payment caused by the Japanese occupation. died from an accident. Crown refused to pay because of the lapse of premium payment

RULING: The beneficiaries are not entitled to recover for non-payment despite the ISSUE: Whether Gonzaga's widow can claim despite the absence of premium payment.
presence of war. Contracts of insurance are contracts of indemnity within the terms and
condition found therein. An insurance company for certain considerations guarantee the HELD: Non-payment of premiums by reason of war puts an end to the contract of
insured against loss or damage as may be stipulated, and when called to pay, the insurer insurance. Time is material and of the essence of the contract. Non-payment at the day
may insist on the fulfillment of said stipulations. Failure of the insured to do so involves absolute forfeiture if such be the terms of the contract. Courts cannot with
disqualifies recovery for the loss. Thus the terms of the policy determines the insurer’s safety vary the stipulation of the parties by introducing equities for the relief of the
liability. Compliance to the terms of the policy is a must as it is a condition precedent to insured against their own negligence. Where the offices of the defendant insurance
the right of recovery. Therefore, from the terms of the policy it is clear that non- company, being an enemy corporation, were ordered closed by the Japanese Military
payment of premium produces avoidance (forfeiture of the policy). Since the year 1917, authorities, but the company opened an office clandestinely for the purpose of receiving
the Philippine law on Insurance was found in Act No. 2427, as amended, and the Civil premiums from policy holders, the failure of the defendant to advice the insured of the
Code. Act No. 2427 was largely copied from the Civil Code of California. And this court defendant's new address did not work as a forfeiture of its right to have the premiums
has heretofore announced its intention to supplement the statutory laws with general satisfied promptly. There is no duty where the law forbids; and there is no obligation
principles prevailing on the subject in the United State. without a corresponding right enjoyed by another.
Mortgagor and Mortgagee 4. Palileo v. Cosio, 97 Phil 919

3. Grepalife v. Court of Appeals, 316 SCRA 677 FACTS: Plaintiff obtained from defendant a loan in the sum of P12,000. To secure the
payment of the aforesaid loan, defendant required plaintiff to sign a document known
FACTS: There was an existing group life insurance executed between Great Pacific Life as “Conditional Sale of Residential Building”, purporting to convey to defendant, with
Assurance (Grepalife) and the Development Bank of the Philippines (DBP). Grepalife right to repurchase, a two-story building of strong materials belonging to plaintiff. This
agreed to insure the lives of eligible housing loan mortgagors of DBP. In November document did not express the true intention of the parties which was merely to place
1983, Wilfredo Leuterio, mortgagor of DBP applied to be a member of the group life said property as security for the payment of the loan. After the execution of the
insurance. He filled out a form where he indicated he never consulted any physician aforesaid document, defendant insured the building against fire for the sum of P15,000,
regarding any illness (heart condition etc) and that he is in good health. He was the insurance policy having been issued in the name of defendant. The building was
eventually included in the group life insurance and he was covered for the amount of his partly destroyed by fire and, after proper demand, defendant collected from the
indebtedness (P86,200.00). In August 1984, Wilfredo died. DBP submitted a death claim insurance company an indemnity of P13,107.00. Plaintiff demanded from defendant
but it was denied by Grepalife as it insisted that Wilfredo actually concealed that he was that she be credited with the necessary amount to pay her obligation out of the
suffering from hypertension at the time of his insurance application. Grepalife relied on insurance proceeds but defendant refused to do so.
the statement made by the doctor who issued Wilfredo’s death certificate wherein it
was stated that Wilfredo’s immediate cause of death was massive cerebral hemorrhage ISSUE: Whether a mortgagor is entitled to the insurance proceeds of the mortgaged
secondary to hypertension or hypertension as a “possible cause of death”. Since property independently insured by the mortgagee?
Grepalife refused to pay the insurance claim filed by DBP, Medarda Leuterio (widow)
sued Grepalife. Grepalife assailed the suit and insisted that Medarda is not a proper HELD: NO. Where a mortgagee, independently of the mortgagor, insures the mortgaged
party in interest. The lower court ruled in favor of Medarda and the court ordered property in his own name and f or his own interest, he is entitled to the insurance
Grepalife to pay the amount of the insurance to DBP. The Court of Appeals affirmed this proceeds in case of loss, but in such case, he is not allowed to retain his claim against
decision in 1993. Grepalife appealed to the Supreme Court. In 1995, pending resolution the mortgagor, but is passed by subrogation to the insurer to the extent of the money
of the case in the SC, DBP foreclosed the property of Medarda. paid. Or, stated in another way, "the mortgagee may insure his interest in the property
independently of the mortgagor. In that event, upon the destruction of the property the
ISSUE: Whether or not DBP has insurable interest as creditor. insurance money paid to the mortgagee will not inure to the benefit of the mortgagor,
and the amount due under the mortgage debt remains unchanged. The mortgagee,
however, is not allowed to retain his claim against the mortgagor, but it passes by
HELD: YES. In this type of policy insurance, the mortgagee is simply an appointee of the subrogation to the insurer, to the extent of the insurance money paid.
insurance fund, such loss-payable clause does not make the mortgagee a party to the
contract. Section 8 of the Insurance Code provides: “Unless the policy provides, where a 5. Saura Import v. Phil. International Surety, 8 SCRA 143
mortgagor of property effects insurance in his own name providing that the loss shall be
payable to the mortgagee, or assigns a policy of insurance to a mortgagee, the insurance FACTS: Saura Import & Export Co Inc., mortgaged to the Phil. National Bank, a parcel of
is deemed to be upon the interest of the mortgagor, who does not cease to be a party
land.
to the original contract, and any act of his, prior to the loss, which would otherwise
The mortgage was amended to guarantee an increased amount, bringing the total
avoid the insurance, will have the same effect, although the property is in the hands of
the mortgagee, but any act which, under the contract of insurance, is to be performed mortgaged debt to P37,000. On the land mortgage is a building owned by Saura Import
by the mortgagor, may be performed by the mortgagee therein named, with the same & Export Co Inc. which was insured with Philippine International Surety (Insurer) even
effect as if it had been performed by the mortgagor.” Petitioner claims that there was before the mortgage contract so it was required to endorse to mortgagee PNB. Barely
no evidence as to the amount of Dr. Leuterio’s outstanding indebtedness to DBP at the 13 days after the issuance of the fire insurance policy, the insurer cancelled it. Notice of
time of the mortgagor’s death. Hence, for private respondent’s failure to establish the the cancellation was given to PNB (mortgagee). But Saura (insured) was not informed.
same, the action for specific performance should be dismissed. Petitioner’s claim is
On 1955, the building and all its contents worth P40,685.69 were burned so Saura filed a
without merit. A life insurance policy is a valued policy. Unless the interest of a person
insured is susceptible of exact pecuniary measurement, the measure of indemnity under claim with the Insurer and mortgagee Bank.
a policy of insurance upon life or health is the sum fixed in the policy. Where the
mortgagee under a mortgage redemption insurance has already foreclosed on the ISSUE: Whether Philippine International Surety should be held liable for the claim
mortgage, it cannot collect the insurance proceeds—the proceeds then rightly belong to because notice to only the mortgagee is not substantial.
the heirs of the mortgagor.
Ruling: YES. Appealed from is hereby reversed. Philippine International Surety Co., Inc., Realizing that it could no longer collect from the insurance company because the same
to pay Saura Import & Export Co., Inc., P29,000. The policy in question does not provide had already folded up, the petitioner directed the collection suit against the private
for the notice, its form or period. The Insurance Law, Act No. 2427, does not likewise respondents whose obligation with the petitioner had long been extinguished. Indeed,
and as found by the lower courts, the petitioner could have collected the insurance
provide for such notice. This being the case, it devolves upon the Court to apply the
proceeds if only it were not negligent.
generally accepted principles of insurance, regarding cancellation of the insurance policy
by the insurer. Actual notice of cancellation in a clear and unequivocal manner, 7. San Miguel v. Law Union Rock, 40 Phil 674
preferably in writing, in view of the importance of an insurance contract, should be
given by the insurer to the insured, so that the latter might be given an opportunity to FACTS: San Miguel Brewery, for the purpose of recovering upon two policies of
obtain other insurance for his own protection. The notice should be personal to the insurance underwritten respectively by Law Union and Rock Insurance Company (Ltd.),
insured and not to and/or through any unauthorized person by the policy. and the "Filipinas" Compania de Seguros, for the sum of P7,500 each, insuring certain
property which has been destroyed by fire. Plaintiff is named as the party assured in the
6. Philippine National Bank v. Court of Appeals, 158 SCRA 201 two policies referred to, but it is alleged in the complaint that said company was in
reality interested in the property which was the subject of insurance in the character of a
FACTS: On January 10, 1963, the private respondents-spouses applied for a retailers’ mortgage creditor only, and that the owner of said property upon the date the policies
loan with the petitioner. The loan which was subsequently approved was secured by a were issued was one D. P. Dunn who was later succeeded as owner by one Henry
chattel mortgage consisting of the verified inventory of stocks in the store of the Harding. Accordingly said Harding was made a defendant, as a person interested in the
private respondents, located at Marahui Street, Zamboanga City. In addition to this, the subject of the litigation.
goods and merchandise, subject matter of the mortgage, were insured with the
Cosmopolitan Insurance Co. in the amount of P4,000.00 with the petitioner as the Harding answered, admitting the material allegations of the complaint and claiming for
beneficiary pursuant to the requirements of the latter. On August 1, 1964, while the himself the right to recover the difference between the plaintiff's mortgage credit and
insurance and the chattel mortgage were still in force, and after the private respondents the face value of the policies. The two insurance companies also answered, admitting in
had paid the petitioner the amount of P1,089.60 as partial payment of the loan in effect their liability to the San Miguel Brewery to the extent of its mortgage credit, but
accordance with the loan agreement, the insured building and merchandise of the denying liability to Harding on the ground that under the contracts of insurance the
private respondents were totally destroyed by fire. The petitioner sent several letters to liability of the insurance companies was limited to the insurable interest of the plaintiff
the insurance company for the purpose of recovering the proceeds of the insurance but therein.
to no avail. Sometime in 1966, the said insurance company became the subject of
liquidation. Seven years after the insured chattels mortgaged were burned, the In this connection it appears that D. P. Dunn, then the owner of the property to which
petitioner filed a complaint for collection against the private respondents. the insurance relates, mortgaged the same to the San Miguel Brewery to secure a debt
of P10,000. In the contract of mortgage Dunn agreed to keep the property insured at his
ISSUE: Whether petitioner can collect against the private respondents. expense to the full amount of its value in companies to be selected by the Brewery
Company and authorized the latter in case of loss to receive the proceeds of the
RULING: The petitioner as the attorney-in-fact of the private respondents and as the insurance and to retain such part as might be necessary to cover the mortgage debt. At
beneficiary of the insurance policy had the obligation to collect the proceeds of the the same time, in order more conveniently to accomplish the end in view, Dunn
policy. The argument of the petitioner to the effect that there is no express provision in authorized and requested the Brewery Company to effect said insurance itself.
the Chattel Mortgage Contract which compels the petitioner to collect the proceeds of Accordingly on the same date Antonio Brias, general manager of the Brewery, made a
the insurance in case of loss is a mere rationalization of one trying hard to put the blame verbal application to the Law Union and Rock Insurance Company for insurance to the
on another for its own fault or negligence. For "under the chattel mortgage covering the extent of P15,000 upon said property. It therefore issued its own policy for P7,500 and
goods offered as security for payment of the loan, the private respondents as procured a policy in a like amount to be issued by the "Filipinas" Compania de
mortgagors constituted and appointed the petitioner as mortgagee their attorney-in- Seguros. Both policies were issued in the name of the San Miguel. The premiums were
fact with full power and authority to collect and receive any interest, income or benefits paid by the Brewery and charged to Dunn. In the month of March of the year 1917 Dunn
produced by the mortgaged property and apply such amount collected and received in sold the insured property to the defendant Henry Harding, but not assignment of the
payment of the interest accruing and of the principal obligation. The petitioner was itself insurance, or of the insurance policies, was at any time made to him.
the beneficiary of the insurance policy to which it was duly indorsed and made payable,
and was in possession thereof."
ISSUE: Whether Harding may collect from the insurance proceeds.
RULING: No. He is not a party to the contracts of insurance and cannot directly maintain the plaintiff had in the warehouse at No. 188 Soler at the time of the fire, not less, but
an action thereon. His claim is merely of an equitable and subsidiary nature and must be rather more, than 6,200 bales of leaf tobacco worth over P300,000, which is of course
made effective, if at all, through the San Miguel Brewery in whose name the contracts more than the sum total of all the insurances taken out with the defendant herein and
are written. Now the Brewery, as mortgagee of the insured property, undoubtedly had the defendants in the three aforementioned cases.
an insurable interest therein; but it could not, in any event, recover upon these policies
an amount in excess of its mortgage credit. In this connection it will be remembered Trial court: It sentences the defendant the Yek Tong Lin Fire and Marine Insurance
that Antonio Brias, upon making application for the insurance, informed the company Company, Ltd., to pay the plaintiff Emilio Gonzales La O, the amount of one hundred
with which the insurance was placed that the Brewery was interested only as a thousand pesos (P100,000), for which it had accepted the insurance on the leaf tobacco
mortgagee. belonging to said plaintiff, damaged by the fire which destroyed the warehouse at No.
188 Soler Street, where it was stored.
It would, therefore, be impossible for the Brewery mortgage on the insured property.
Sections 16 of the Insurance Act (Act No. 2427) declared that "the measure of an Defendant duly appealed from this judgment, alleging that the trial court erred in
insurable interest in property is the extent to which the insured might be damnified by making reference to the settlement arrived at by the plaintiff and other insurance
loss or injury thereof"; while in the other it is stated that "the insurance shall be applied companies, and in declaring that the only question involved in the case is whether or
exclusively to the proper interest of the person in whose name it is made unless not the tobacco damaged by the fire is worth at least P290,000.
otherwise specified in the policy" (sec. 50).
ISSUE: Whether said goods were worth what the plaintiff claims, that is, about equal to
the amount for which they were insured in the four above mentioned assurance
8. Gonzales Lao v. Yek Tong Lin Fire G.R. No. 33131
companies, including the defendant in this case?
FACTS: Plaintiff demands P290,000 from the defendant assurance companies, alleging
RULING: Yes. Since the settlement between the plaintiff and the other defendant
that to be the amount of the insurance on his leaf tobacco which was damaged by the
companies was reached after the plaintiff had presented his evidence, and as those
fire that destroyed the warehouse, Manila, where it was stored, on January 11, 1922.
three cases were tried jointly with the instant case, there is no valid reason why the trial
The plaintiff's claim against the herein defendant, the Yek Tong Lin Fire & Marine
court should not refer to it in deciding this case.
Insurance Co. being for P100,000, and against the defendants in the three other cases,
for P190,000. Other defendant companies offered to compromise with him by paying
The court's holding here assigned as error, granting there were other incidental matters
eighty-five per cent of his claim against them. Said defendants had in their answer
to be decided by the court, does not in itself constitute a reversible error. Defendant
raised the question of warranties, providing that the building used for the effects
contends that the plaintiff cannot recover under the policy as he has failed to prove that
insured would not be occupied by any other lessee, nor would be used for the deposit of
the Bank of the Philippine Islands, to whom the policy was made payable, no longer has
other goods, without the consent of said defendants. Inasmuch as the latter alleged in
any rights and interests. Defendant did not in its answer allege defect of parties’
their answer that the owner of the burnt building had leased the warehouse to several
plaintiff, and, besides, it does not appear that the plaintiff ceded to the bank all his
persons for the storage of sundry articles, the plaintiff had to accept the proposed
rights or interests in the insurance, the note attached to the policies merely stating:
compromise, and in consequence thereof, the three cases aforesaid were dismissed. In
"There shall be paid to the Bank of the Philippine Islands an indemnity for any loss
this case, plaintiff refused to accept the compromise which, in the same terms as those
caused by fire, according to the interest appearing in its favor."
made by the defendants in the three cases mentioned.
The fact that the plaintiff himself presented in evidence the policies mortgaged to the
Plaintiff: Defendant did not, nor could, raise the question of warranties heretofore
Bank of the Philippine Islands gives rise to the presumption that the debt thus secured
mentioned for the simple reason that it was the defendant itself, as owner, who had
has been paid. That defendant had knowledge of the existence of other policies
leased the building which later was destroyed by fire, to another person after having
obtained by the plaintiff from other insurance companies, is specifically shown by the
already ceded a portion of it to said plaintiff. Plaintiff has conclusively shown [by the
defendant's answer wherein it alleges, by way of special defense, the fact that there
Official Register and the Official Guide, furnished by the Bureau of Internal Revenue; the
exist other policies issued by the companies mentioned therein.
Stock Book for recording the quantity of tobacco kept by the plaintiff and presented as
part of the testimony of witnesses Claveria, Bonete, and Leoncio Jose; the testimony of
Estanislao Lopez, Inspector of Internal Revenue, and the latter's report submitted to the
Collector of Internal Revenue; invoices of stock damaged by the fire; and by the
testimony of Clemente Uson who went over the plaintiff's books as auditor and public
accountant, that:]
Insurable Interest: (Read Insurance Code, Title 3, Sec. 10-26); De Leon p. 85-130] The existence of an insurable interest gives a person the legal right to insure the subject
matter of the policy of insurance. Section 10 of the Insurance Code indeed provides that
Definition every person has an insurable interest in his own life. Section 19 of the same code also
states that an interest in the life or health of a person insured must exist when the
1. Lalican v. Insular Life, 597 SCRA 159 insurance takes effect, but need not exist thereafter or when the loss occurs. In the
instant case, Eulogios death rendered impossible full compliance with the conditions for
FACTS: Violeta is the widow of the deceased Eulogio C. Lalican (Eulogio). During his reinstatement of Policy. True, Eulogio, before his death, managed to file his Application
lifetime, Eulogio applied for an insurance policy with Insular Life. Insular Life, through for Reinstatement and deposit the amount for payment of his overdue premiums and
Josephine Malaluan, its agent in Gapan City, issued in favor of Eulogio a Policy, which interests thereon with Malaluan; but Policy No. 9011992 could only be considered
contained a 20-Year Endowment Variable Income Package Flexi Plan worth P500,000.00, reinstated after the Application for Reinstatement had been processed and approved by
with two riders valued at P 500,000.00 each. Thus, the value of the policy amounted to Insular Life during Eulogios lifetime and good health.
P1,500,000.00. Violeta was named as the primary beneficiary. Under the terms of the
policy, Eulogio was to pay the premiums on a quarterly basis in the amount of 8,062.00, The only recognized exceptions to the doctrine of immutability and unalterability are
payable every 24 April, 24 July, 24 October and 24 January of each year, until the end of the correction of clerical errors, the so called nunc pro tunc entries, which cause no
the 20-year period of the policy. According to the Policy Contract, there was a grace prejudice to any party, and void judgments.
period of 31 days for the payment of each premium subsequent to the first. If any
premium was not paid on or before the due date, the policy would be in default, and if 2. Insular Life v. Ebrado
the premium remained unpaid until the end of the grace period, the policy would
automatically lapse and become void. Eulogio paid the premiums due on 24 July 1997 Section 50 of the Insurance Act which provides that “(t)he insurance shall be applied
and 24 October 1997. However, he failed to pay the premium due on 24 January 1998, exclusively to the proper interest of the person in whose name it is made” cannot be
even after the lapse of the grace period of 31 days. The policy, therefore, lapsed and validly seized upon to hold that the same includes the beneficiary. The word “interest”
became void. Eulogio submitted to the Cabanatuan District Office of Insular Life, highly suggests that the provision refers only to the “insured” and not the beneficiary,
through Malaluan, on 26 May 1998, an Application for Reinstatement of Policy, together since a contract of insurance is personal in character. Otherwise, the prohibitory laws
with the amount of P 8,062.00 to pay for the premium due on 24 January 1998. In a against illicit relationships especially on property and descent will be rendered nugatory,
letter dated 17 July 1998, Insular Life notified Eulogio that his Application for as the same could easily be circumvented by modes of insurance.
Reinstatement could not be fully processed because, although he already deposited
P8,062.00 as payment for the 24 January 1998 premium, he left unpaid the overdue Insular life v. Ebrado: In essence, a life insurance policy is not different from from a civil
interest thereon amounting to P322.48. Thus, Insular Life instructed Eulogio to pay the donation insofar as the beneficiary is concerned. Both are founded upon the same
amount of interest and to file another application for reinstatement. On 17 September consideration: liberality. A beneficiary is like a donee, because from the premiums of the
1998, Eulogio went to Malaluans house and submitted a second Application for policy which the insured pays out of liberality, the beneficiary will receive the proceeds
Reinstatement of Policy, including the amount of P17,500.00, representing payments for or profits of said insurance. As a consequence, the proscription in Article 739 of the new
the overdue interest. As Malaluan was away on a business errand, her husband received Civil Code should equally operate in life insurance contracts. The mandate of Article
Eulogios second Application for Reinstatement and issued a receipt for the amount 2012 cannot be laid aside: any person who cannot receive a donation cannot be named
Eulogio deposited. A while later, on the same day, 17 September 1998, Eulogio died of as beneficiary in the life insurance policy of the person who cannot make the donation.
cardio-respiratory arrest secondary to electrocution. Under American law, a policy of life insurance is considered as a testament and in
construing it, the courts will, so far as possible treat it as a will and determine the effect
ISSUE: Whether or not Eulogio had an existing insurable interest in his own life until the of a clause designating the beneficiary by rules under which wills are interpreted.
day of his death in order to have the insurance policy validly reinstated.
3. El Oriente v. Posadas, 54 Phil 147
RULING: No. An insurable interest is one of the most basic and essential requirements in
an insurance contract. In general, an insurable interest is that interest which a person is FACTS: Plaintiff is a domestic corporation duly organized and existing under and by
deemed to have in the subject matter insured, where he has a relation or connection virtue of the laws of the Philippine Islands, having its principal office in Manila and that
with or concern in it, such that the person will derive pecuniary benefit or advantage the defendant is the duly appointed, qualified and acting Collector of Internal Revenue
from the preservation of the subject matter insured and will suffer pecuniary loss or of the Philippine Islands.
damage from its destruction, termination, or injury by the happening of the event
insured against.
Plaintiff, in order to protect itself against the loss that it might suffer by reason of the Thus, as to the point before us, it was made clear, when not only in the part of the law
death of its manager, A. Velhagen, who had had more than thirty-five (35) years of concerning individuals were exemptions provided for beneficiaries, but also in the part
experience in the manufacture of cigars in the Philippine Islands, and whose death concerning corporations, specific reference was made to the exemptions in favor of
would be a serious loss to the plaintiff, procured from the Manufacturers Life Insurance individuals, thereby making the same applicable to corporations. This was
Co., of Toronto, Canada, thru its local agent E.E. Elser, an insurance policy on the life of authoritatively pointed out and decided by the United States Supreme Court in the case
the said A. Velhagen for the sum of $50,000, United States currency, designated itself as of United States vs. Supplee-Biddle Hardware Co. which involved facts quite similar to
the sole beneficiary of said policy on the life of its said manager and paid from its funds those before us.
all the insurance premiums due thereon. Plaintiff charged as expenses of its business all
the said premiums and deducted the same from its gross incomes as reported in its The Court believes that this fact signifies that when the plaintiff received P104,957.88
annual income tax returns, which deductions were allowed by the defendant upon a from the insurance on the life of its manager, it thereby realized a net profit in this
showing made by the plaintiff that such premiums were legitimate expenses of its amount. It is true that the Income Tax Law, in exempting individual beneficiaries, speaks
(plaintiff's) business. of the proceeds of life insurance policies as income, but this is a very slight indication of
legislative intention. In reality, what the plaintiff received was in the nature of an
Upon the death of said A. Velhagen in the year 1929, the plaintiff received all the indemnity for the loss which it actually suffered because of the death of its manager.
proceeds of the said life insurance policy, aggregating P104,957.88. Over the protest of
the plaintiff, which claimed exemption under section 4 of the Income Tax Law, the Therefore, the purport of the stipulated facts, considering the uncertainty of Philippine
defendant Collector of Internal Revenue assessed and levied the sum of P3,148.74 as law, and considering the lack of express legislative intention to tax the proceeds of life
income tax on the proceeds of the insurance policy, which tax the plaintiff paid under insurance policies paid to corporate beneficiaries, particularly when in the exemption in
instant protest. favor of individual beneficiaries in the chapter on this subject, the clause is inserted
"exempt from the provisions of this law," we deem it reasonable to hold the proceeds of
ISSUE: Whether the proceeds of insurance taken by a corporation on the life of an the life insurance policy in question as representing an indemnity and not taxable
important official to indemnify it against loss in case of his death, are taxable as income income.
under the Philippine Income Tax Law.
BENEFICIARY
RULING: The Income Tax Law for the Philippines is Act No. 2833, as amended. In chapter
I On Individuals, is to be found section 4 which provides that, "The following incomes 1. Vda. De Consaguera v. GSIS, 37 SCRA 316
shall be exempt from the provisions of this law: (a) The proceeds of life insurance
policies paid to beneficiaries upon the death of the insured ... ." Section 10, as amended, FACTS: Jose Consuegra was employed as a shop foreman of the Office of the District
in Chapter II On Corporations, provides that, There shall be levied, assessed, collected, Engineer in Surigao Del Norte. When he was still alive, he contracted two marriages:
and paid annually upon the total net income received in the preceding calendar year First, Rosario Diaz with whom he had two children namely, Jose Consuegra Jr. and Pedro
from all sources by every corporation ... a tax of three per centum upon such income ...
but both predeceased him. His second marriage was with Basilia Berdin with whom he
." Section 11 in the same chapter, provides the exemptions under the law, but neither
here nor in any other section is reference made to the provisions of section 4 in Chapter had seven children. This was contracted in good faith while the first marriage subsisted.
I. Under the view we take of the case, it is sufficient for our purposes to direct attention Being a GSIS member when he died, the proceeds of his life insurance were paid by the
to the anomalous and vague condition of the law. It is certain that the proceeds of life GSIS to Berdin and her children who were the beneficiaries named in the policy. Since
insurance policies are exempt. It is not so certain that the proceeds of life insurance he was in the gov’t service for over 22 years, he was entitled to retirement insurance
policies paid to corporate beneficiaries upon the death of the insured are likewise benefits, for which no beneficiary was designated. Both families filed their claims with
exempt. But at least, it may be said that the law is indefinite in phraseology and does the GSIS, which ruled that the legal heirs were Diaz who is entitled to one-half of the
not permit us unequivocally to hold that the proceeds of life insurance policies received
retirement benefits and Berdin and her children were entitled to the remaining half,
by corporations constitute income which is taxable. The situation will be better
elucidated by a brief reference to laws on the same subject in the United States. The each to receive an equal share. Berdin went to CFI on appeal. CFI affirmed GSIS
Income Tax Law of 1916 extended to the Philippine Legislature, when it came to enact decision.
Act No. 2833, to copy the American statute. Subsequently, the Congress of the United
States enacted its Income Tax Law of 1919, in which certain doubtful subjects were ISSUE: To whom should the retirement insurance benefits be paid?
clarified.
RULING: Both families are entitled to half of the retirement benefits. The beneficiary By its terms, the insurance company agreed to insure the life of Hilario to be paid him
named in the life insurance does NOT automatically become the beneficiary in the on February 1, 1930, or if the insured should die before said date, then to his wife,
retirement insurance. When Consanguera, during the early part of 1943, or before should she survive him. The policy did not include any provision reserving to the insured
1943, designated his beneficiaries in his life insurance, he could NOT have intended the right to change the beneficiary. On the date the policy was issued, Andrea Zialcita
those beneficiaries of his life insurance as also the beneficiaries of his retirement was the lawful wife of Hilario. Towards the end of the year 1919, she was convicted of
insurance because the provisions on retirement insurance under the GSIS came about the crime of adultery, and a decree of divorce was issued, which had the effect of
only when CA 186 was amended by RA 660 on June 18, 1951. completely dissolving their marital bonds. In 1922, Hilario formally notified the Sun Life
that he had revoked his donation in favor of Andrea Zialcita, and that he had designated
Sec. 11(b) clearly indicates that there is need for the employee to file an application for in her stead his present wife, Adela Garcia de Gercio, as the beneficiary of the policy. He
retirement insurance benefits when he becomes a GSIS member and to state his requested the insurance company to eliminate Andrea Zialcita as beneficiary. This, the
beneficiary. The life insurance and the retirement insurance are two separate and insurance company has refused and still refuses to do. The judgment of the trial court
distinct systems of benefits paid out from 2 separate and distinct funds. was in favor of Gercio, and ordered the defendant company to eliminate from the
insurance policy the name of Andrea Zialcita as beneficiary.
In the case of the proceeds of a life insurance, the same are paid to whoever is named
the beneficiary in the life insurance policy. As in the case of life insurance provided for in ISSUE: Whether the insured — the husband — has the power to change the beneficiary
the Insurance Act (Act 2427, as amended), the beneficiary in a life insurance under the — the former wife — and to name instead his actual wife, where the insured and the
GSIS may not necessarily be an heir of the insured. The insured in a life insurance may beneficiary have been divorced and where the policy of insurance does not expressly
designate any person as beneficiary unless disqualified to be so under the provisions of reserve to the insured the right to change the beneficiary.
the Civil Code. And in the absence of any beneficiary named in the life insurance policy,
the proceeds of the insurance will go to the estate of the insured. The Supreme Court, in RULING: No. The wife has an insurable interest in the life of her husband. The
construing the rights of two women who were married to the same man, held “that beneficiary has an absolute vested interest in the policy from the date of its issuance and
since the defendant’s first marriage has not been dissolved or declared void the conjugal delivery. So when a policy of life insurance is taken out by the husband in which the wife
partnership established by that marriage has not ceased. is named as beneficiary, she has a subsisting interest in the policy. If the policy contains
no provision authorizing a change of beneficiary without the beneficiary’s consent, the
Nor has the first wife lost or relinquished her status as putative heir of her husband insured cannot make such change.
under the new Civil Code, entitled to share in his estate upon his death should she
survive him. Consequently, whether as conjugal partner in a still subsisting marriage or Accordingly, it is held that a life insurance policy of a husband made payable to the wife
as such putative heir she has an interest in the husband’s share in the property in as beneficiary, is the separate property of the beneficiary and beyond the control of the
dispute.” And with respect, to the right of the second wife, this Court observed that husband. As to the effect produced by the divorce, the Philippine Divorce Law, Act No.
although the second marriage can be presumed to be void ab initio as it was celebrated 2710, merely provides in section 9 that the decree of divorce shall dissolve the
while the first marriage was still subsisting, still there is need for judicial declaration of community property as soon as such decree becomes final. Therefore, in the absence of
its nullity. And inasmuch as the conjugal partnership formed by the second marriage was a statute to the contrary, that if a policy is taken out upon a husband’s life the wife is
dissolved before judicial declaration of its nullity, “the only just and equitable solution in named as beneficiary therein, a subsequent divorce does not destroy her rights under
this case would be to recognize the right of the second wife to her share of one-half in the policy. It will be proper, in the first place, to ascertain what is an insurable interest.
the property acquired by her husband, and consider the other half as pertaining to the
conjugal partnership of the first marriage.” It is well settled that a man has an insurable interest in his own life and in that of his
wife and children; a woman in the life of her husband; and the creditor in the life of his
2. Gercio v. Sun Life Insurance, 48 Phil 53 debtor. Indeed it may be said generally that any reasonable expectation of pecuniary
benefit or advantage from the continued life of another creates an insurable interest in
FACTS: On January 29, 1910, Sun Life Assurance Co. of Canada issued an insurance such life. And there is no doubt that a man may effect an insurance on his own life for
policy on the life of Hilario Gercio. the benefit of a relative or friend.
It is indeed the general rule that a policy, and the money to become due under it, belong, ISSUE: Whether the insurer may refuse to grant the loan application and the surrender
the moment it is issued, to the person or persons named in it as the beneficiary or of the policy claimed by a father-guardian in behalf of his minor son absent a court
beneficiaries, and that there is no power in the person procuring the insurance, by any authority in a guardianship proceeding.
act of his, by deed or by will, to transfer to any other person the interest of the person
named. The rights of a beneficiary in an ordinary life insurance policy become vested
upon the issuance of the policy, and can thereafter, during the life of the beneficiary, be HELD: YES. The proposed transactions in question (policy loan and surrender of policy)
constitute acts of disposition or alienation of property rights and not merely of
defeated only as provided by the terms of the policy.
management or administration because they involve the incurring or termination of
contractual obligations. It appearing that the minor beneficiary's vested interest or right
on the policy exceeds two thousand pesos (P2,000.00), the plaintiffs should have, but,
3. Nario v. Philam Life, 20 SCRA 434 had not, filed a guardianship bond and a formal application or petition for
guardianship, Under Section 7, Rule 93 of the Revised Rules of Court. Hence, plaintiffs
FACTS: Mrs. Alejandra Santos-Mario was issued by the Philippine American Life cannot possibly exercise the powers vested on them, as legal administrators of their
Insurance Co. a life insurance policy under a 20-year endowment plan, with a face value child's property, under articles 320 and 326 of the Civil Code. The result would be the
of P5,000.00, on June 12, 1959. She designated her husband, Delfin Nario, and their same even if we regarded the interest of the ward to be worth less than P2,000.00.
unemancipated minor son, Ernesto Nario, as her irrevocable beneficiaries. Sometime in While the father or mother would in such event be exempt from the duty of filing a
June 1963, Mrs. Nario applied for a loan with the Insurance Company for the purpose of bond, and securing judicial appointment, still the parent's authority over the estate of
using the proceeds for the school expenses of her minor son, Ernesto Nario. The the ward as a legal-guardian would not extend to acts of encumbrance or disposition, as
application had the written signature and consent of Delfin Nario as the father-guardian distinguished from acts of management or administration.
of minor son and as the legal administrator of the minor's properties. The Insurance
Company denied the application, alleging that the written consent for the minor son The vested interest or right of the beneficiaries in a life insurance policy should be
must not only be given by his father as legal guardian but it must also be authorized by measured on its full face value and not on its cash surrender value, for, in case of death
the court in a competent guardianship proceeding. After the denial of the loan of the insured, said beneficiaries are paid on the basis of its face value and, in case the
application, Mrs. Nario signified her decision to surrender her policy to the Insurance insured should discontinue paying premiums, the beneficiaries may continue paying the
Company, demanded its cash value which then amounted to P520.00. The Insurance same and they are entitled to automatic extended term or paidup insurance options.
Company also denied the surrender of the policy on the same grounds as above.
4. Philam Life v. Pineda, 175 SCRA 416
Hence, Mrs. Alejandra Santos-Nario and her husband brought suit against the Philippine
American Life Insurance Co. in the CFI, seeking to compel the latter (defendant) to grant FACTS: On January 15, 1968, private respondent procured an ordinary life insurance
their policy loan application and/or to accept the surrender of said policy in exchange for policy from the petitioner company and designated his wife and children as irrevocable
its cash value. beneficiaries of said policy. Under date February 22, 1980 private respondent filed a
Defendant Insurance Company answered the complaint, virtually admitting its material petition which was docketed as Civil Case No. 9210 of the then Court of First Instance of
allegations, but it set up the affirmative defense that under article 320 in relation to Rizal to amend the designation of the beneficiaries in his life policy from irrevocable to
article 326 of the Civil Code, mere written consent given by the father-guardian, for and revocable. Petitioner, filed its Comment and/or Opposition to Petition. When the
in behalf of the minor son, without any court authority therefore, was not a sufficient petition was called for hearing on March 19, 1980, the respondent presiding Judge of
compliance of the law. CFI agreed with the defendant company and said that the the then Court of First Instance of Rizal, Pasig Branch XXI, denied petitioner's Urgent
consent given by the father-guardian was without the requisite court authority hence, Motion to Reset the Hearing, thus allowing the private respondent to adduce evidence,
plaintiffs' complaint was dismissed. Petitioner appealed directly to this Court, the consequence of which was the issuance of the order granting the petition to amend
contending that the minor's interest amounted to only one-half of the policy's cash the designation of the beneficiaries from irrevocable to revocable. Petitioner promptly
surrender value of P520.00. Further, under Section 2, Rule 96 of the Revised Rules of filed a Motion for Reconsideration but the same was denied in an Order June 10, 1980.
Court, payment of the ward's debts is within the powers of the guardian, where no Hence, this petition
realty is involved. Hence, there is no reason why the father may not validly agree to the
proposed transaction on behalf of the minor without need of court authority. ISSUES: Whether the irrevocable beneficiaries herein, one of whom is already deceased
while the other are all minors, could validly give consent to the change or amendment in
the designation of the irrevocable beneficiaries.
RULINGS: NO. The applicable law in the instant case is the Insurance Act, the policy Gaisano is a customer and dealer of the products of IMC and LSPI. On February 25, 1991,
having been procured in 1968. Under the said law, the beneficiary designated in a life the Gaisano Superstore Complex in Cagayan de Oro City, owned by petitioner, was
insurance contract cannot be changed without the consent of the beneficiary because consumed by fire. Included in the items lost or destroyed in the fire were stocks of
he has a vested interest in the policy. In this regard, it is worth noting that the ready-made clothing materials sold and delivered by IMC and LSPI. Insurance of America
Beneficiary Designation Indorsement in the policy states that the designation of the filed a complaint for damages against Gaisano. It alleges that IMC and LSPI were paid for
beneficiaries is irrevocable. It is only with the consent of all the beneficiaries that any their claims and that the unpaid accounts of petitioner on the sale and delivery of ready-
change or amendment in the policy concerning the irrevocable beneficiaries may be made clothing materials with IMC was P2,119,205.00 while with LSPI it was
legally and validly effected. Both the law and the policy do not provide for any other P535,613.00.
exception, thus, abrogating the contention of the private respondent that said
designation can be amended if the Court finds a just, reasonable ground to do so. The RTC rendered its decision dismissing Insurance's complaint. It held that the fire was
purely accidental; that the cause of the fire was not attributable to the negligence of the
Similarly, the alleged acquiescence of the six (6) children beneficiaries of the policy (the petitioner. Also, it said that IMC and LSPI retained ownership of the delivered goods and
beneficiary-wife predeceased the insured) cannot be considered an effective ratification must bear the loss. The CA rendered its decision and set aside the decision of the RTC. It
to the change of the beneficiaries from irrevocable to revocable. Indubitable is the fact ordered Gaisano to pay Insurance the P 2 million and the P 500,000 the latter paid to
that all the six (6) children named as beneficiaries were minors at the time, for which IMC and Levi Strauss. Hence this petition.
reason, they could not validly give their consent. Neither could they act through their
father insured since their interests are quite divergent from one another. The insured ISSUE: Whether a fire insurance policy on book debts covers the unpaid accounts of IMC
can do nothing to divest the beneficiary of his rights without his consent. He cannot and LSPI since such insurance applies to loss of the ready-made clothing materials sold
assign his policy, nor even take its cash surrender value without the consent of the and delivered to petitioner.
beneficiary. Neither can the insured's creditors seize the policy or any right thereunder.
The insured may not even add another beneficiary because by doing so, he diminishes RULING: No. Nowhere is it provided in the questioned insurance policies that the
the amount which the beneficiary may recover and this he cannot do without the subject of the insurance is the goods sold and delivered to the customers and dealers of
beneficiary's consent. Therefore, the parent-insured cannot exercise rights and/or the insured. Thus, what were insured against were the accounts of IMC and LSPI with
privileges pertaining to the insurance contract, for otherwise, the vested rights of the petitioner which remained unpaid 45 days after the loss through fire, and not the loss or
irrevocable beneficiaries would be rendered inconsequential. destruction of the goods delivered.

Insurable interest in property Insurance Section 13 of our Insurance Code defines insurable interest as "every interest in
property, whether real or personal, or any relation thereto, or liability in respect
1. Gaisano Cagayan v. Ins, Co. of North America, 490 SCRA 286 thereof, of such nature that a contemplated peril might directly damnify the insured."
Parenthetically, under Section 14 of the same Code, an insurable interest in property
FACTS: IMC and Levi Strauss (Phils.) Inc. (LSPI) separately obtained from respondent fire may consist in: (a) an existing interest; (b) an inchoate interest founded on existing
insurance policies with book debt endorsements. The insurance policies provide for interest; or (c) an expectancy, coupled with an existing interest in that out of which the
coverage on "book debts in connection with ready-made clothing materials which have expectancy arises.
been sold or delivered to various customers and dealers of the Insured anywhere in the
Philippines." Anyone has an insurable interest in property who derives a benefit from its existence or
The policies defined book debts as the "unpaid account still appearing in the Book of would suffer loss from its destruction.
Account of the Insured 45 days after the time of the loss covered under this Policy." The Indeed, a vendor or seller retains an insurable interest in the property sold so long as he
policies also provide for the following conditions: has any interest therein, in other words, so long as he would suffer by its destruction, as
where he has a vendor's lien. In this case, the insurable interest of IMC and LSPI pertain
1. Warranted that the Company shall not be liable for any unpaid account in respect of to the unpaid accounts appearing in their Books of Account 45 days after the time of the
the merchandise sold and delivered by the Insured which are outstanding at the date of loss covered by the policies. Petitioner bears the loss under Article 1504 (1) of the Civil
loss for a period in excess of six (6) months from the date of the covering invoice or Code. Moreover, it must be stressed that the insurance in this case is not for loss of
actual delivery of the merchandise whichever shall first occur. goods by fire but for petitioner's accounts with IMC and LSPI that remained unpaid 45
days after the fire. Accordingly, petitioner's obligation is for the payment of money.
2. Warranted that the Insured shall submit to the Company within twelve (12) days after Where the obligation consists in the payment of money, the failure of the debtor to
the close of every calendar month all amount shown in their books of accounts as unpaid make the payment even by reason of a fortuitous event shall not relieve him of his
and thus become receivable item from their customers and dealers. liability.
The rationale for this is that the rule that an obligor should be held exempt from liability RULING: The Court ruled that there is a prima facie showing in Lepanto’s complaint and
when the loss occurs thru a fortuitous event only holds true when the obligation pleadings that it is a real party in interest under the policies and that it has a cause of
consists in the delivery of a determinate thing and there is no stipulation holding him action against the petitioners as insurers. This holding is based (1) on the stipulation in
liable even in case of fortuitous event. It does not apply when the obligation is pecuniary the two policies that it has an interest therein and (2) on the facts that it was the shipper
in nature. There is no proof of full settlement of the insurance claim of LSPI; no (and presumably the owner) of the insured cargoes, that the shipments were
subrogation receipt was offered in evidence. Thus, there is no evidence that respondent undertaken in accordance with the instructions of the insurer’s marine surveyor and
has been subrogated to any right which LSPI may have against petitioner. Failure to that it was Lepanto that filed the corresponding claim with the adjuster when the
substantiate the claim of subrogation is fatal to petitioner’s case for recovery of the cargoes were damaged. Commercial Union rejected the claims, not on those grounds,
amount of P535,613.00. but because "both cargoes were inherently vicious". To say that Lepanto has no interest
under the policies would render meaningless the said stipulation in its favor. To say that
Lepanto as shipper of the insured property had no proprietary interest therein before its delivery at
2. Commercial Union Assurance v. Lepanto, 113 SCRA 752 Asarco’s wharf in Tacoma is to imply that the insured property was res nullius.

FACTS: This is a marine insurance case. Lepanto Consolidated Mining Company alleged 3. Harding v. Commercial Union Assurance, 38 Phil 464
in its complaint that on November 8 and 23, 1971 it shipped (for smelting) copper ore
concentrates on board the vessels M/S Hermosa and M/S General Aguinaldo from Poro FACTS: In February 1916, Mrs. Harding applied for car insurance for a Studebaker she
Point, San Fernando, La Union to Tacoma, Washington, U.S.A. The first shipment is received as a gift from her husband. She was assisted by Smith, Bell and Co which was
known as No. 167 and the other shipment as Nos. 168 and 168-A. The copper ore the duly authorized representative (insurance agent) of Commercial Union in the
concentrates were stored on board the carrying vessels under the supervision and Philippines. The car’s value was estimated with the help of an experienced mechanic
approval of a marine surveying firm designated by the insurer. American Smelting and (Mr. Server) of the Luneta Garage. The car was bought by Mr. Harding for P2,800.00.
Refining Co., Ltd. (Asarco) was the consignee. The ore was to be discharged at the wharf The mechanic, considering some repairs done, estimated the value to be at P3,000.00.
of Asarco’s smelter at Tacoma. The shipments were covered by two "all risks" marine This estimated value was the value disclosed by Mrs. Harding to Smith, Bell and Co. She
insurance policies issued to Asarco by North British & Mercantile Insurance Company also disclosed that the value was an estimate made by Luneta Garage (which also acts as
Limited, a subsidiary of Commercial Union Assurance Company Limited. The first policy an agent for Smith, Bell and Co). In March 1916, a fire destroyed the Studebaker. Mrs.
was for US$4,509,014 or 80% of the agreed total value of US$5,636,268 while the Harding filed an insurance claim but Commercial Union denied it as it insisted that the
second policy was for US$6,230,591.03 or 80% of the agreed total value of representations and averments made as to the cost of the car were false; and that said
US$7,788,233.79. The 20% balance was covered by insurance policies issued by Malayan statement is a warranty. Commercial Union also stated that the car does not belong to
Insurance Co., Inc. Both policies contain this stipulation: "It is hereby noted and agreed Mrs. Harding because such a gift [from her husband] is void under the Civil Code.
that Lepanto Consolidated Mining Co. have (has) an interest on this Policy". From the
opening clause of the policies, it may be inferred that Asarco and all persons having an ISSUE: Whether statements signed by the insured and made out by the person
interest in the shipments were covered by the insurance. Because the two shipments authorized to solicit the insurance shall bind the insurer (Commercial).
were damaged in transit, Lepanto filed claims under the policies. Commercial Union
Assurance and North British denied the claims. Lepanto filed a complaint in the Court of RULING: Yes. Commercial is liable. Where it appears that the proposal form, while
First Instance of Rizal, Pasig Branch 22, against Commercial Union Assurance and North signed by the insured was made out by the person authorized to solicit the insurance
British wherein it prayed that they be ordered to pay Lepanto. CFI dismissed the (Luneta and Smith Bell) the facts stated in the proposal, even if incorrect, will not be
complaint for lack of cause of action. Court of Appeals reversed the order of dismissal. regarded as warranted by the insured, in the absence of willful misstatement. Under
such circumstances, the proposal is to be regarded as the act of the insurer. Section 163
CA denied the motion for reconsideration filed by Commercial Union Assurance and of the Insurance Law (Act No. 2427) provides that “the effect of a valuation in a policy of
North British. Herein petitioner filed a special civil action of certiorari in this Court fire insurance is the same as in a policy of marine insurance.” By the terms of section
wherein they alleged that the Court of Appeals acted without jurisdiction in entertaining 149 of the Act cited, the valuation in a policy of marine insurance is conclusive if the
Lepanto’s appeal. The certiorari petition was treated as an appeal. As directed, the insured had an insurable interest and was not guilty of fraud. The court, therefore, of
parties filed their briefs. the opinion and hold that plaintiff was the owner of the automobile in question and had
an insurable interest therein. A wife has an insurable interest in the property given to
ISSUE: Whether Lepanto can claim ownership of the cargo covered by the marine her by her husband. There was no fraud on her part in procuring the insurance.
insurance policies; Whether it has personality to sue; The valuation in a policy of fire insurance is conclusive in the absence of fraud. The
valuation of the automobile, for the purposes of the insurance, is binding upon the
defendant corporation.
4. Lampano v. Jose, 30 Phil 537 A building contractor is not obligated to surrender to the owner or her grantees any part
of the proceeds of a policy insuring his own interest exclusively and paid for by him, for
the mere reason that, at the time of the fire, the amount of the policy exceeds that still
FACTS: Defendant, Mariano R. Barretto, constructed a house for the other defendant,
due him on the construction price.
Placida A. Jose, on the subject land described for the agreed price of P6,000. Subsequent
thereto and Placida sold the house to the plaintiff, Antonina Lampano, for the sum of
P6,000. On March 22, 1913, the house was destroyed by fire. At the time of the fire Insurable interest of carrier of depository
Antonina still owed Placida the sum of P2,000, evidenced by a promissory note, and
Placida still owed Mariano the sum of P2,000. After the completion of the house and 1. Lopez v. Del Rosario, 44 Phil 98
sometime before it was destroyed, Mariano took out an insurance policy upon it in his
own name, with the consent of Placida, for the sum of P4,000. After its destruction, he FACTS: Mrs. Del Rosario, was the owner of a bonded warehouse situated in the City of
collected P3,600 from the insurance company, having paid in premiums the sum of Manila. She was engaged in the business of a warehouse keeper, and stored copra and
P301.50. The plaintiff alleged in her complaint that there was a verbal agreement other merchandise in the said building. Among the persons who had copra deposited in
between her and Placida A. Jose, at the time of the purchase and sale of the house, to the Del Rosario warehouse was Froilan Lopez, the holder of fourteen warehouse receipts
the effect that the latter agreed to deliver to her the insurance policy on the building; in his own name, and the name of Elias T. Zamora. The warehouse receipts, or
that she did not learn that the policy was in the name of Barretto until after the fire; and negotiable warrants, or quedans (as they are variously termed) of Lopez named a
that neither Placida nor Mariano has any right to the insurance or to the money declared value of P107,990.40.
received therefrom. She prayed for judgment against each of them for the amount of
the insurance collected. Judgment was entered against Barretto and in favor of Placida Mrs. Del Rosario secured insurance on the warehouse and its contents with the National
for the sum of P1,298.50, being the difference between the amount collected by Insurance Co., Inc., the Commercial Union Insurance Company, the Alliance Insurance
Barretto on the insurance and the amount yet due him for the construction of the Company, the South British Insurance Co., Ltd., and the British Traders Insurance Co.,
house, including the premiums paid. Judgment was also entered in favor of the Ltd., in the amount of P404,800. All the policies were in the name of Sra. Benita
defendant, Placida, against the plaintiff for the sum of P2,000, being the balance of the Quiogue de V. del Rosario, with the exception of one of the National Insurance
purchase price of the house. The plaintiff was authorized to offset this judgment against Company, Inc., for P40,000, in favor of the Compañia Coprera de Tayabas. The
her for P2,000 by the P2,000 which the court declared had been paid the defendant, warehouse of Mrs. Del Rosario and its contents were destroyed by fire on June 6, 1920.
Placida, by Barretto out of the insurance money. The warehouse was a total loss, while of the copra stored therein, only an amount
equal to P49,985 was salvaged. When Bayne, a fire loss adjuster, failed to effect a
CA: The court found that there was no privity of contract between the plaintiff and the settlement between the Insurance companies and Del Rosario, the latter authorized
defendant Barretto. In consequence, no judgment was entered in favor of the plaintiff Atty. Fisher to negotiate with the Companies. An agreement was reached to submit the
against the defendant. It entered a judgment against Barretto and in favor of Placida for matter to arbitration. The arbitrators in their report allowed Mrs. Del Rosario P363,610,
the sum of P1,298.50. This was done upon the theory that the insurance policy was held which, with the addition of the money received from the salvaged copra amounting to
in trust for Placida, and that any balance, resulting after deducting the amount owing P49,985, and interest, made a total of P414,258, collected by her from the companies.
upon the construction contract and paid for premiums, belonged to her. The claims by different people who had stored copra in the warehouse were settled
with the exception of Friolan Lopez. Ineffectual attempts by Mrs. Del Rosario to effect a
compromise with Lopez first for P71,994, later raised to P72,724, and finally reduced to
ISSUE: Does the plaintiff have any right to recover from Barretto any portion of the
P17,000, were made. Lopez stubbornly contended that he should receive not a centavo
insurance money?
less than P88,595.43.

RULING: No. In the case at bar Barretto assumed the responsibility for the insurance. ISSUE: Whether the plaintiff Lopez is entitled to receive insurance benefits even if he did
The premiums, as we have indicated, were paid by him without any agreement or right not request or know of the insurance, and did not ratify it before the payment of the
to recoup the amount paid therefor should no loss result to the property. It would not, loss.
therefore, be in accordance with the law and his contractual obligations to compel him
to account for the insurance money, or any part thereof, to the plaintiff, who assumed RULING: Yes. The SC noted in two documents, one the agreement for arbitration, and
no risk whatever. That Barretto had an insurable interest in the house, we think there the other the statement of claim of Mrs. Del Rosario against the insurance companies,
can be no question. He construed the building, furnishing all the materials and supplies, she acknowledged her responsibility to the owners of the stored merchandise, against
and insured it after it had been completed. A building contractor has an insurable risk of loss by fire. The award of the arbitrators covered not Mrs. Del Rosario's
interest in the completed building pending the payment of the construction price. warehouse alone but the products stored in the warehouse by Lopez and others.
The law is that a policy effected by bailee and covering by its terms his own property ISSUE: Whether a lessee has an insurable interest in the equipment and motor vehicles
and property held in trust; inures, in the event of a loss, equally and proportionately to leased, and what is the measure of its insurable interest.
the benefit of all the owners of the property insured. Even if one secured insurance
covering his own goods and goods stored with him, and even if the owner of the stored RULING: Yes. Lim, as a lessee, has an insurable interest in the equipment and motor
goods did not request or know of the insurance, and did not ratify it before the payment vehicles leased. The stipulation in Section 14 of the lease contract, that the equipment
of the loss, yet it has been held by a reputable court that the warehouseman is liable to shall be insured at the cost and expense of the lessee against loss, damage, or
the owner of such stored goods for his share. destruction from fire, theft, accident, or other insurable risk for the full term of the lease,
is a binding and valid stipulation. Petitioner, as a lessee, has an insurable interest in the
Plaintiff's rights to the insurance money have not been forfeited by failure to pay the equipment and motor vehicles leased. Section 17 of the Insurance Code provides that
insurance provided for in the warehouse receipts. A preponderance of the proof does the measure of an insurable interest in property is the extent to which the insured might
not demonstrate that the plaintiff ever ordered the cancellation of his insurance with be damnified by loss or injury thereof. It cannot be denied that JVL will be directly
the defendant. Nor is it shown that the plaintiff ever refused to pay the insurance when damnified in case of loss, damage, or destruction of any of the properties leased.
the bills were presented to him, and that notice of an intention to cancel the insurance
was ever given the plaintiff. Warranty: In the financial lease agreement, FEB did not assume responsibility as to the
quality, merchantability, or capacity of the equipment. This stipulation provides that, in
Agency: The remaining contention of the defendant that the plaintiff cannot claim the case of defect of any kind that will be found by the lessee in any of the equipment,
benefits of the agency without sharing in the expenses, is well taken. Although the recourse should be made to the manufacturer. “The financial lessor, being a financing
plaintiff did not expressly authorize the agreement to submit the matter to arbitration, company, i.e., an extender of credit rather than an ordinary equipment rental company,
yet on his own theory of the case, Mrs. Del Rosario was acting as his agent in securing does not extend a warranty of the fitness of the equipment for any particular use. Thus,
insurance, while he benefits from the amicable adjustment of the insurance claims. the financial lessee was precisely in a position to enforce such warranty directly against
Under the facts stated in the opinion, that defendant acted as agent of plaintiff in taking the supplier of the equipment and not against the financial lessor.
out insurance on the contents of the bodega. The agency can be deduced from the
warehouse receipts, the insurance policies, and the circumstances surrounding the Purpose of FEB: The basic purpose of a financial leasing transaction is to enable the
transaction. prospective buyer of equipment, who is unable to pay for such equipment in cash in one
lump sum, to lease such equipment in the meantime for his use, at a fixed rental
sufficient to amortize at least 70% of the acquisition cost with the expectation that at
Measure of Insurable Interest the end of the lease period the financial lessee will be able to pay any remaining balance
of the purchase price.
1. Ong Lim Sing Jr. v. FEB Leasing, 524 SCRA 333
2. Philam Health Systems v. Court of Appeals, 379 SCRA 356
FACTS: FEB Leasing and Finance Corporation (FEB) leased equipment and motor vehicles
to JVL Food Products with a monthly rental of P170,494. At the same date, Vicente Ong FACTS: In 1988, Ernani Trinos applied for a health care insurance under the Philamcare
Lim Sing, Jr. (Lim) executed an Individual Guaranty Agreement with FEB to guarantee the Health Systems, Inc. He was asked if he was ever treated for high blood, heart trouble,
prompt and faithful performance of the terms and conditions of the lease agreement. diabetes, cancer, liver disease, asthma, or peptic ulcer; he answered no. His application
JVL defaulted in the payment of the monthly rentals resulting to arrears of was approved and it was effective for one year. His coverage was subsequently renewed
P3,414,468.75 and refused to pay despite demands. FEB filed a complaint for damages twice for one year each. While the coverage was still in force in 1990, Ernani suffered a
and replevin against JVL and Lim. JVL and Lim admitted the existence of the lease heart attack for which he was hospitalized. The cost of the hospitalization amounted to
agreement but asserted that it is in reality a sale of equipment on installment basis, with P76,000.00. Julita Trinos, wife of Ernani, filed a claim before Philamcare for the latter to
FEB acting as the financier pay the hospitalization cost. Philamcare refused to pay as it alleged that Ernani failed to
disclose the fact that he was diabetic, hypertensive, and asthmatic. Julita ended up
RTC: It rendered an order ruling that it is a sale on installment and the FEB elected full paying the hospital expenses. Ernani eventually died. In July 1990, Julita sued
payment of the obligation so for the unreturned units and machineries the JVL and Lim Philamcare for damages. Philamcare alleged that the health coverage is not an
are jointly and severally liable to pay. insurance contract; that the concealment made by Ernani voided the agreement.

CA: It granted FEB appeal that it is a financial lease agreement under Republic Act (R.A.) ISSUE: Whether a health care agreement is not an insurance contract hence the
No. 8556 and ordered JVL and Lim jointly and severally to pay P3,414,468.75 “incontestability clause” under the Insurance Code does not apply.
RULING: No. Petitioner claimed that it granted benefits only when the insured is alive When CKS learned of the insurance earlier procured by the Cha spouses (without its
during the one-year duration. It contended that there was no indemnification unlike in consent), it wrote the insurer (United) a demand letter asking that the proceeds of the
insurance contracts. It supported this claim by saying that it is a health maintenance insurance contract (between the Cha spouses and United) be paid directly to CKS, based
organization covered by the DOH and not the Insurance Commission. Lastly, it claimed on its lease contract with the Cha spouses. United refused to pay CKS. Hence, the latter
that the Incontestability clause is inapplicable because two-year and not one-year filed a complaint against the Cha spouses and United. The RTC rendered a decision
effectivity period is required. ordering United to pay CKS the amount of P335,063.11 and the Cha spouses to pay
P50,000.00 as exemplary damages, P20,000.00 as attorney’s fees and costs of suit. On
In this case, the husband’s health was the insurable interest. The health care agreement appeal, the Court of Appeals affirmed the trial court decision, deleting however the
was in the nature of non-life insurance, which is primarily a contract of indemnity. The awards for exemplary damages and attorney’s fees.
provider must pay for the medical expenses resulting from sickness or injury. Section 3
of the Insurance Code states that any contingent or unknown event, whether past or ISSUE: Whether or not the CKS has insurable interest because the spouses Cha violated
future, which may damnify a person having an insurable interest against him, may be the stipulation on the lease contract.
insured against. Every person has an insurable interest in the life and health of himself.

To whom it must be paid: Petitioner alleges that respondent was not the legal wife of RULING: NO. Under Sec. 18 of the Insurance Code of the Philippines which provides that
the deceased member considering that at the time of their marriage, the deceased was “No contract or policy of insurance on property shall be enforceable except for the
previously married to another woman who was still alive. The health care agreement is benefit of some person having an insurable interest in the property insured”.
in the nature of a contract of indemnity. Hence, payment should be made to the party
who incurred the expenses. It is not controverted that respondent paid all the hospital A non-life insurance policy such as the fire insurance policy taken by petitioner-spouses
and medical expenses. She is therefore entitled to reimbursement. The records over their merchandise is primarily a contract of indemnity. Insurable interest in the
adequately prove the expenses incurred by respondent for the deceased’s property insured must exist at the time the insurance takes effect and at the time the
hospitalization, medication and the professional fees of the attending physicians. loss occurs. The basis of such requirement of insurable interest in property insured is
(measure of insurable interest) based on sound public policy: to prevent a person from taking out an insurance policy
on property upon which he has no insurable interest and collecting the proceeds of
Fraud: Concealment as a defense for the health care provider or insurer to avoid liability said policy in case of loss of the property. In such a case, the contract of insurance is a
is an affirmative defense and the duty to establish such defense by satisfactory and mere wager which is void under Section 25 of the Insurance Code. In Sec. 25 of the same
convincing evidence rests upon the provider or insurer; The liability of the health care Code states, “Every stipulation in a policy of Insurance for the payment of loss, whether
provider attaches once the member is hospitalized for the disease or injury covered by the person insured has or has not any interest in the property insured, or that the policy
the agreement or whenever he avails of the covered benefits which he has prepaid. shall be received as proof of such interest, and every policy executed by way of gaming
or wagering, is void”. The automatic assignment of the policy to CKS under the provision
Effect when no insurable interest exists of the lease contract previously quoted is void for being contrary to law and/or public
policy. The proceeds of the fire insurance policy thus rightfully belong to the spouses
1. Spouses Cha v. Court of Appeals, 277 SCRA 690 Nilo Cha and Stella Uy-Cha (herein co-petitioners.) The insurer (United) cannot be
compelled to pay the proceeds of the fire insurance policy to a person (CKS) who has no
FACTS: Spouses Nilo Cha and Stella Uy-Cha, as lessees, entered into a lease contract insurable interest in the property insured.
with CKS Development Corporation, as lessor, on 5 October 1988. One of the
stipulations of the 1 year lease contract states that “The LESSEE shall not insure against Measure of interest: Also under Sec. 17 of the same Code provides that the measure of
fire the chattels, merchandise, textiles, goods and effects placed at any stall or store or an insurable interest in property is the extent to which the insured might be damnified
space in the leased premises without first obtaining the written consent and approval of by loss of injury thereof. Hence, the automatic assignment of the policy to CKS under
the LESSOR. If the LESSEE obtain(s) the insurance thereof without the consent of the the provision of the lease contract previously quoted is void for being contrary to law
LESSOR then the policy is deemed assigned and transferred to the LESSOR for its own and/or public policy. The proceeds of the fire insurance policy thus rightfully belong to
benefit” Notwithstanding the above stipulation in the lease contract, the Cha spouses the spouses. The liability of the Cha spouses to CKS for violating their lease contract in
insured against loss by fire their merchandise inside the leased premises for that Cha spouses obtained a fire insurance policy over their own merchandise, without
P500,000.00 with the United Insurance Co., Inc. without the written consent of CKS. On the consent of CKS, is a separate and distinct issue which we do not resolve in this
the day that the lease contract was to expire, fire broke out inside the leased premises. case.
2. Garcia v. Hongkong Fire and Marine Insurance, G.R. No. 20341 RULING: No. It clearly appears that where the word "merchandise" was written in the
letter of August 6th above quoted, some other word had been previously written and
erased, and the word "merchandise" was the written, as it now appears. Although the
FACTS: Plaintiff alleged that it entered into a contract with the defendant Domingo
policy was in possession of the Bank, the defendant had among its own records all of the
Garcia, then a merchant and owner of a bazaar known as "Las Novedades", whereby it
data and information upon which the policy was issued, and, as a matter of fact, its
insured his merchandise in the sum of P15,000 at a premium of P300 per annum; that in
agents knew or should have known the kind of property insured. It is possible that when
consideration of such premium, the defendant issued its fire insurance a policy in favor
the Bank wrote the letter, it knew of the error in the issuance of the policy. But that is a
of the plaintiff, not on the merchandise in the building, but on the building which
matter of inference or conjecture only. Outside of the appearance of the letter itself,
contained the merchandise; that for such reason the policy does not contain the true
there is no evidence that the Bank had any acknowledge of the error. Garcia had his
agreement and intent of the parties; that the plaintiff was not the owner of, and did not
dealings with the officials of the branch Bank at Legaspi where he was doing business as
have any interest in, the building; and that the policy was so issued through error,
a merchant, of which the officials of that Bank had knowledge. Under such facts, the
carelessness and negligence of the defendant.
presumption of knowledge, if any, on the part of the Bank would be that the policy
was on the merchandise. Garcia was a well known merchant, and his merchandise was
That Garcia executed a mortgage to the plaintiff Bank on the merchandise insured by the in the building described in the policy. Under these circumstances it seems clear and
defendant, and that with the consent of the defendant, the plaintiff endorsed the policy manifest that the insured, as well as the manager of the National Bank at Legaspi, who
to the Bank; and while the policy was in force and effect, a fire took place which was interested in the policy, have been in the belief that it was not the building but the
destroyed the merchandise in the building of the value of P20,000, together with the merchandise that was insured, for the reason that none of them paid attention to the
building itself; that demand was made upon the defendant for the payment of P15,000, context of the policy.
as provided for in the policy, and that payment was refused.
When insurable interest must exist
It appears that the policy was in the English language, of which the plaintiff Garcia is
ignorant. When he received it he noticed that the amount P15,000 was correct, and 1. Tai Tong Chuache v. Insurance Commission, 158 SCRA 366
never personally made a further investigation. He was the exclusive owner of the
merchandise in the building which, at the time of the fire, was of the probable value of FACTS: Azucena Palomo obtained a loan from Tai Tong Chuache Inc. in the amount of
P20,000. He did not own or claim any interest in the building. Desiring to have his P100,000.00. To secure the payment of the loan, a mortgage was executed over the
merchandise insured for P15,000, he wrote a letter to "El Pilar," requesting that firm to land and the building in favor of Tai Tong Chuache & Co. Arsenio Chua, representative of
have it insured, as a result of which, the policy in question was issued and delivered to Tai Tong Chuache & Co. insured the latter's interest with Travellers Multi-Indemnity
him, and it was issued on the building with Garcia did not own, and did not cover the Corporation for P100,000.00 (P70,000.00 for the building and P30,000.00 for the
merchandise which he did own. Desiring to obtain a loan from the Philippine National contents thereof). Pedro Palomo secured a Fire Insurance Policy covering the building
Bank, Garcia later delivered and assigned the policy to the plaintiff Bank as collateral for P50,000.00 with respondent Zenith Insurance Corporation. On July 16, 1975, another
security for a loan. Fire Insurance was procured from respondent Philippine British Assurance Company,
covering the same building for P50,000.00 and the contents thereof for P70,000.00. The
Where later Garcia assigned the policy to a bank as collateral security for the payment building and the contents were totally razed by fire.
of a loan, and the bank notified the insurance company in writing "that the merchandise
insured by you against fire in favor" of G, giving the amount and number of the policy, Based on the computation of the loss, respondents, Zenith Insurance, Phil. British
and requested that the insurance company should make the proper endorserment in Assurance and S.S.S. Accredited Group of Insurers, paid their corresponding shares of
favor of the bank, and the insurance company acknowledged receipt of the letter and the loss. Complainants were paid the following: P41,546.79 by Philippine British
complied with the request, and nothing was said about the mistake, and where about six Assurance Co., P11,877.14 by Zenith Insurance Corporation, and P5,936.57 by S.S.S.
months later both the building and the merchandise were destroyed by fire. It is Group of Accredited Insurers. Demand was made from respondent Travellers Multi-
contended by the defendant that when the letter was written, the Bank, which then had Indemnity for its share in the loss but the same was refused. Hence, complainants
the possession of the policy, knew that it covered the building and did not insure the demanded from the other three (3) respondents the balance of each share in the loss in
merchandise. That, having such knowledge, it was the duty of the Bank to notify the the amount of P30,894.31 (P5,732.79-Zenith Insurance: P22,294.62, Phil. British: and
defendant, and having failed to do so, it cannot now contend that the policy was issued P2,866.90, SSS Accredited) but the same was refused, hence, this action.
through a mistake.

ISSUE: Whether it was the duty of the Bank to notify the defendant that the policy
covered the building and did not insure the merchandise.
In their answers, respondents denied liability on the ground that the claim of the Change in interest in property
complainants had already been waived, extinguished or paid. Both companies set up
counterclaim in the total amount of P 91,546.79. SSS Accredited Group of Insurers San Miguel v. Law Union Rock, supra
informed the Commission that the claim of complainants for the balance had been paid As regards Harding, an additional insuperable obstacle is found in the fact that the
in the amount in full. Travellers Insurance, on its part, admitted the issuance of a Policy ownership of the property had been charged, prior to the loss, without any
and alleged defenses that the Fire Policy covering the furniture and building of corresponding change having been effected in the policy of insurance. In section 19 of
complainants was secured by a certain Arsenio Chua and that the premium due on the the Insurance Act the Court finds it stated that "a change of interest in any part of a
fire policy was paid by Arsenio Chua. Tai Tong Chuache & Co. also filed a complaint in thing insured unaccompanied by a corresponding change of interest in the insurance,
intervention claiming the proceeds of the fire Insurance Policy issued by respondent suspends the insurance to an equivalent extent, until the interest in the thing and the
Travellers Multi-Indemnity. Respondent Insurance Commission likewise dismissed interest in the insurance are vested in the same person." Again in section 55 it is
spouses Palomos' complaint on the ground that the insurance policy subject of the declared that "the mere transfer of a thing insured does not transfer the policy, but
complaint was taken out by Tai Tong Chuache & Company, for its own interest only as suspends it until the same person becomes the owner of both the policy and the thing
mortgagee of the insured property and thus complainant as mortgagors of the insured insured."
property have no right of action against the respondent. Tai Tong Chuache filed a
motion for reconsideration but it was likewise denied hence, the present petition.

ISSUE: Whether Tai Tong had insurable interest over the proceeds of the policy taken by
mortgagor Palomo over the building and its contents.

RULING: Yes. The record of the case shows that the petitioner to support its claim for
the insurance proceeds offered as evidence the contract of mortgage which has not
been cancelled nor released. It has been held in a long line of cases that when the
creditor is in possession of the document of credit, he need not prove non-payment for
it is presumed. The validity of the insurance policy taken by petitioner was not assailed
by private respondent. Moreover, petitioner's claim that the loan extended to the
Palomos has not yet been paid was corroborated by Azucena Palomo who testified that
they are still indebted to herein petitioner. Each party must prove his own affirmative
allegations. Petitioner who is claiming a right over the insurance must prove its case.
Likewise. respondent insurance company to avoid liability under the policy by setting up
an affirmative defense of lack of insurable interest on the part of the petitioner must
prove its own affirmative allegations.

Party in interest: Petitioner being a partnership may sue and be sued in its name or by
its duly authorized representative. The fact that Arsenio Lopez Chua is the
representative of petitioner is not questioned. Petitioner s declaration that Ar senio
Lopez Chua acts as the managing partner of the partnership was corroborated by
respondent insurance company. Thus Chua as the managing partner of the partnership
may execute all acts of administration including the right to sue debtors of the
partnership in case of their failure to pay their obligations when it became due and
demandable.

Вам также может понравиться